Вы находитесь на странице: 1из 139

LEGAL TECHNIQUES AND LOGIC JUDGE ALGODON

I. INTRODUCTION

A. History of Logic B. Civil Law vs. Common Law Tradition C. The Role of Logic in Law

A. History of Logic

The founder of Logic is Aristotle of Stagira in Thrace (384-322B.C.), son of the physician NIchomachus and became Plato s student at 18 years of age and studied with him for 20 years. Shortly afterward in 343 B.C., became a tutor for 3 years to the 13- year old Alexander, who was to rule the world as Alexander the Great. In 335 B. C., he founded his own school, the Peripathetic School of philosophers.

Aristotle s work on logic is found in his Organon (meaning the method or organ of investigation) which consisted of a number of his writings, including the following: On Interpretation -dealt with the structure of logical proposition; Prior Analytics-the doctrine of syllogism; Posterior Analytics-the logic of science and the applications of the syllogism; Topics-the logic of argument based on probable truths; Sophistical Test-dealt with logical fallacies.

Logical reasoning makes us certain that our conclusions are true and this provides us with accepted scientific proofs of universally valid propositions or statements.

B. Civil Law vs. Common Law Tradition

COMMON LAW: That which derives its force and authority from the universal consent and immemorial practice of the people. The system of jurisprudence that originated in England and which was latter adopted in the U.S. that is based on precedent instead of statutory laws. Traditional law of an area or region; also known as case law. The law created by judges when deciding individual disputes or cases. The body of law which includes both the unwritten law of England and the statutes passed before the settlement of the United States.

In the common law, civil law is the area of laws and justice that affect the leg al status of individuals. Civil law, in this sense, is usually referred to in comparison to criminal law, which is th at body of law involving the state against individuals (including incorporated organizations) where the state relie s on the power given it by statutory law.

CIVIL LAW:

Civil law may also be compared to military law, administrative law and constitut ional law (the laws governing the political and law making process), and international law. Where there are le gal options for causes of action by individuals within any of these areas of law, it is thereby civil law.

Civil law courts provide a forum for deciding disputes involving torts (such as accidents, negligence, and libel), contract disputes, the probate of wills, trusts, property disputes, admi nistrative law,commercial law, and any other private matters that involve private parties and organizations inc luding government departments. An action by an individual (or legal equivalent) against the attorn ey general is a civil matter, but when the state, being represented by the prosecutor for the attorney general, or some other agent for the state, takes action against an individual (or legal equivalent including a gover nment department), this is public law, not civil law.

The objectives of civil law are different from other types of law. In civil law: a. there is the attempt to right a wrong, b. honor an agreement, or c. settle a dispute. d. If there is a victim, they get compensation, and the person who is the cause of the wrong pays, this being a civilized form of, or legal alternative to, revenge. e. If it is an equity matter, there is often a pie for division and it gets allo cated by a process of civil law, possibly invoking the doctrines of equity. f. In public law, the objective is usually deterrence, and retribution.

An action in criminal law does not necessarily preclude an action in civil law i n common law countries, and may provide a mechanism for compensation to the victims of crime. Such a situati on occurred when O.J. Simpson was ordered to pay damages for wrongful death after being acquitted of t he criminal charge of murder.

Civil law in common law countries usually refers to both common law and the law of equity, which while now merged in administration, have different traditions, and have historically opera ted to different doctrines, although this dualism is increasingly being set aside so there is one coherent b ody of law rationalized around common principles of law.

C. Role of Logic in Law

Regardless of the professions we are in, we always use logic. We use it when we make decisions or when we try to influence the decisions of others or when we are engaged in argumentation and debate. A lawyer presents his arguments using the principle of logic to prove the tenabi lity of his position, otherwise, he will send his client to jail. Everybody uses logic since everyone possesses r eason.

II. REASONING A. Basic Concepts

1. What is Logic 2. Propositions and Sentences 3. Arguments, Premises and Conclusions 4. More Complex Arguments 5. Recognizing Arguments 6. Deduction and Induction 7. Validity and Truth 8. Arguments and Explanations

B. Analyzing and Diagramming Arguments

C. Problem Solving

1. What is Logic

1. Reasoning conducted or assessed according to strict principles of validity: " experience is a better guide to this than deductive logic". 2. A particular system or codification of the principles of proof and inference: "Aristotelian logic".

2. Propositions and Sentences

Proposition refers to either the "content" or "meaning" of a meaningful declarat ive sentence. The meaning of a proposition includes having the quality or property of being either true or false

Propositional logic largely involves studying logical connectives such as the wo rds and and or and the rules determining the truth-values of the propositions they are used to join, as

well as what these rules mean for the validity of arguments, and such logical relationships between statements as being consistent or inconsistent with one another, as well as logical properties of propositions

Sentence . Sentence logic deals with sentences of a natural language that are either true or false . Sentence logic ignores the internal structure of simple sentences . Sentence logic is concerned with sentences which are compounded in a certain w ay. . A primary goal of sentence logic is to enable the evaluation of a certain clas s of arguments in natural language . In an argument, a sentence that is the argument s con

3. Arguments, Premises and Conclusions

Argument is a connected series of statements intended to establish a definite pr oposition. ...an argument is an intellectual process... contradiction is just the automatic gainsaying of any thing the other person says.

An argument is a deliberate attempt to move beyond just making an assertion. Whe n offering an argument, you are offering a series of related statements which represent an attempt to su pportthat assertion to give others good reasons to believe that what you are asserting is true rather t han false. Here are examples of assertions: 1. 2. 3. 4. Shakespeare wrote the play Hamlet. The Civil War was caused by disagreements over slavery. God exists. Prostitution is immoral.

Sometimes you hear such statements referred to as propositions. Technically spea king, a proposition is the informational content of any statement or assertion. To qualify as a proposition , a statement must be capable of being either true or false 3 major components of Argument: 1. Premise 2. Inference 3. Conclusion Premises are statements of (assumed) fact which are supposed to set forth the re asons and/or evidence for believing a claim. The claim, in turn, is the conclusion: what you finish with a t the end of an argument. When an argument is simple, you may just have a couple of premises and a conclusion: 1. Doctors earn a lot of money. (premise) 2. I want to earn a lot of money. (premise) 3. I should become a doctor. (conclusion) Inferences are the reasoning parts of an argument. Conclusions are a type of inf erence, but always the final inference. Usually an argument will be complicated enough to require inferences linking the premises with the final conclusion: 1. 2. 3. 4. 5. Doctors earn a lot of money. (premise) [FACTUAL] With a lot of money, a person can travel a lot. (premise) [FACTUAL] Doctors can travel a lot. (inference, from 1 and 2) I want to travel a lot. (premise) I should become a doctor. (from 3 and 4)

2 Types of Claims: 1. Factual Claim 2. Inferential Claim

- it expresses the idea that some matter of fact is related to the sought-after conclusion. This is the attempt to link the factual claim to the conclusion in such a way as to support the conc lusion. The third statement above is an inferential claim because it infers from the previous two statements that doctors can travel a lot. Without an inferential claim, there would be no clear connection between the pre mises and the conclusion. It is rare to have an argument where inferential claims play no role. Sometimes you will come across an argument where inferential claims are needed, but missing you won t be able to see the connection from factual claims to conclusion and will have to ask for them. 5. Recognizing Arguments we use the term "argument" to mean a set of propositions in which some propositi ons--the premises--are asserted as support or evidence for another--the conclusion. The author doesn't just tell us something that he takes to be true; he also pres ents reasons intended to convince us that it is true. This intention is usually signaled by certain indicator words. The following is a list of the more common indicator words: Premise Indicators Conclusion Indicators Since Therefore Because Thus As So For Consequently Given that As a result Assuming that It follows that Inasmuch as

Hence The reason is that Which means that In view of the fact that Which implies that

6. Deduction and Induction Deduction: A deductive argument claims that its premises make its conclusion certain.

This deductive argument is valid because the conclusion follows with certainty if the premises are true. There is no possible way for the premises to be true and yet the conclusion false Example: All mammals have lungs. All whales are mammals. Therefore all whales have lungs.

tree.jpg A valid deductive argument with true premises is a sound argument. A sound argum ent is often called a proof, but this term can be misleading. If the premises themselves are absolutely certain, then a sound argument does indeed offer proof, as in the below example: 1. All bachelors are unmarried. 2. All bachelors are male. 3. Therefore all bachelors are unmarried males. The premises are certain here because they are true by definition, and the argum ent is sound, so the conclusion is proven

Inductive: Inductive arguments do not try to establish their conclusions with certainty. In stead, an inductive argument claims that its premises make the conclusion probable. Inductive arguments canno t be valid or invalid. Instead, they are weak or strong, better or worse. And even when the premises ar e true and provide very strong support for the conclusion, the conclusion cannot be certain. The stronge st inductive argument is not as conclusive as a sound deductive argument.

Example: Most corporation lawyers are conservatives. Betty Morse is a corporation lawyer. Therefore Betty Morse is a conservative.

7. Validity and Truth

Validity: Deductive arguments may be either valid or invalid. If an argument is valid, and its premises are true, the conclusion must be true: a valid argument cannot have true premises and a false conclusion.

The validity of an argument depends, however, not on the actual truth or falsity of its premises and conclusions, but solely on whether or not the argument has a valid logical form. The validity of an argument is not a guarantee of the truth of its conclusion. A valid argument may have false premises and a false conclusion. The corresponding conditional of a valid argument is a necessary truth (true in all possible worlds) and so the conclusion necessarily follows from the premises, or follows of logical necessit y. The conclusion of a valid argument is not necessarily true, it depends on whether the premises are true. T he conclusion of a valid argument need not be a necessary truth: if it were so, it would be so independen tly of the premises. For example: Some Greeks are logicians; therefore, some logicians are Greeks. Valid argument; it would be self-contradictory to admit that some Greeks are log icians but deny that some (any) logicians are Greeks.

All Greeks are human and all humans are mortal; therefore, all Greeks are mortal . : Valid argument; if the premises are true the conclusion must be true.

Some Greeks are logicians and some logicians are tiresome; therefore, some Greek s are tiresome. Invalid argument: the tiresome logicians might all be Romans (for example).

Either we are all doomed or we are all saved; we are not all saved; therefore, w e are all doomed. Valid argument; the premises entail the conclusion. (Remember that this does not mean the conclusion has to be true; it is only true if the premises are true, which they may not be!)

Premise 1: Some men are hawkers. Premise 2: Some hawkers are rich. Conclusion: S ome men are rich. This argument is invalid. There is a way where you can determine whether an argu ment is valid, give a counter-example with the same argument form.

Counter-Example: Premise 1: Some people are herbivores. Premise 2: Some herbivor es are zebras. Conclusion: Some people are zebras. (This is obviously false.) Note that the counter-example follows the P1. Some x are y. P2. Some y are z. C. Some x are z. format. We can now conclude that the hawker argument is invalid. Arguments can be invalid for a variety of reasons. There are well-established pa tterns of reasoning that render arguments that follow them invalid; these patterns are kno wn as logical fallacies.

Truth: 1. Coherence - Is a statement true when it aligns with, is consistent with and d oesn't contradict other true statements? 2. Correspondence - Is a statement true when it corresponds to something in the real world? 3. Foundationalism - Can certain statements be asserted as true in and of themse lves, self-evidently? Will truth then deductively follow from these assumptions? 4. Pragmatic accounts - Is a statement true when it proves to be useful or pract ical? () 5. Consensus - Is a statement true when enough people argue or believe that it i s true? ()

6. Deflationism - Is truth not an actual property of a statement at all, but som ething else? Can 'true' or 'the truth' ever be predicated in a meaningful, non-redundant way? ()

8. Argument and Explanation Argument Types: 1. Deductive Argument - asserts that the truth of the conclusion is a logical consequence of the premi ses 2. Inductive Argument - asserts that the truth of the conclusion is supported by the premises

Argument is an attempt to persuade someone of something, by giving reasons or ev idence for accepting a particular conclusion. The general structure of an argument in a natural languag e is that of premises (typically in the form of propositions, statements or sentences) in support of a claim: the conclusion. Many arguments can also be formulated in a formal language. An argument in a formal language sh ows the logical form of the natural language arguments obtained by its interpretations.* arguments obtained by its interpretations.*

Distinguish arguments from explanations.

Argument Explanation

(1) expresses an

inference does not usually express an inference

(2) offers evidence, grounds or reasons offers an account why

(3) goes from well known statements to statements less well known gives less well known statements why a better known statement is true

(4) draws a logical connection between statements describes a

causal connection

(5) has the purpose to establish the truth of a statement has the purpose to give an account of something

Definition of an Argument and an Explanation Argument has a number of different definitions. Essentially, it is a line of logi c that is presented in order to support the veracity of a statement. Argument has combative connotations, but an argument does not have to be belligerent. Explanation is used to clarify and explicate a statement. Its aim is to make the listener understand the statement rather than persuade him to accept a certain point of view. Example of an Argument and an Explanation Argument one person wants to convince the other person that it is going to snow t omorrow. He will cite predictions from the weather station, as well as the clouds visible on the horiz on, the damp chill in the air, and the squirrels furiously hiding their nuts. Explanation one both people agree is it going to snow tomorrow because, they say, there is a cold front coming in and the air feels damp. In both cases, the example of snow is used, but note that the argument is trying to convince someone of the truth of their statement, whereas with the explanation, it is not a matter of if the statement is true, but why it is true. Uses of Arguments and Explanations Arguments arguments are used in a variety of professional and academic applicatio ns. For instance, a debate club will take on both sides of an argument and strive to prove each one is right. Arguments are also used by lawyers to convince the jury of the defendant s guilt or innocence. Diplom ats will approach a negotiating table with a certain argument in mind. Entrepreneurs will present po tential backers with an argument in support of their business model. Explanations are used all the time in the classroom to put across new items to st udents. Giving directions is a form of explanation. You will also find explanations included with most new pu rchases, especially those with some assembly required. When the aforementioned entrepreneur is presenting an argument about his business model, he may be asked to explain how it all works.

Summary: 1.Arguments and explanations are both used to get the point across when speaking or writing. 2.Arguments are persuasive and seek to make people understand that something is true, whereas explanations start with the assumption of truthfulness and tell why or how the s tatement has come into being. 3.Both arguments and explanations have wide application in education and busines s, but arguments are used

for persuasion and explanations are used for clarification. Explanation: An explanation is a set of statements constructed to describe a set of facts whi ch clarifies the causes, context, and consequences of those facts.

In scientific research, explanation is one of the purposes of research, e.g., ex ploration and description. Explanation is a way to uncover new knowledge, and to report relationships among different aspects of studied phenomena. 4 Types of Explanation:

1. A thing's material cause is the material of which it consists. (For a table, that might be wood; for a statue, that might be bronze or marble.) 2. A thing's formal cause is its form, i.e. the arrangement of that matter. 3. A thing's efficient or moving cause[4] is "the primary source of the change o r rest." An efficient cause of x can be present even if x is never actually produced and so should not be confused with a sufficient cause.[5] (Aristotle argues that, for a table, this would be t he art of table-making, which is the principle guiding its creation.)[2] 4. A thing's final cause is its aim or purpose. That for the sake of which a thi ng is what it is. (For a seed, it might be an adult plant. For a sailboat, it might be sailing. For a bal l at the top of a ramp, it might be coming to rest at the bottom.)

III. LANGUAGE

A. Uses of Language

1. Three Basic Functions of Language 2. Discourse Serving Multiple Functions 3. Forms of Discourse 4. Emotive Words 5. Kinds of Agreement and Disagreement 6. Emotively Neutral Language

1. Three Basic Functions of Language

1. Informative language function: essentially, the communication of information.

a. The informative function affirms or denies propositions, as in science or the statement of a fact. b. It is used to describe the world or reason about it (e.g.., whether a state o f affairs has occurred or not or what might have led to it). c. These sentences have a truth value; that is, the sentences are either true or false (recognizing, of course, that we might not know what that truth value is). Hence , they are important for logic.

2. Expressive language function: reports feelings or attitudes of the writer (or speaker), or of the subject, or evokes feelings in the reader (or listener).

a. Poetry and literature are among the best examples, but much of, perhaps most of, ordinary language discourse is the expression of emotions, feelings or attitudes . b. Two main aspects of this function are generally noted: (1) evoking certain fe elings and (2) expressing feelings. c. Expressive discourse, qua expressive discourse, is best regarded as neither t rue or false. E.g., Shakespeare's King Lear's lament, "Ripeness is all!" or Dickens' "I t was the best of

times, it was the worst of times; it was the age of wisdom; it was the age of fo olishness " Even so, the "logic" of "fictional statements" is an interesting area of inquiry .

3. Directive language function: language used for the purpose of causing (or pre venting) overt action. a. The directive function is most commonly found in commands and requests. b. Directive language is not normally considered true or false (although various logics of commands have been developed). c. Example of this function: "Close the windows." The sentence "You're smoking i n a nonsmoking area, although declarative, can be used to mean "Do not smoke in this area."

2. Discourse Serving Multiple Functions

Almost any ordinary communication will probably exhibit all three uses of langua ge. Thus a poem, which may be primarily expressive, also may have a moral and thus also be directive. And, of course, a poem may contain a certain amount of information as well. Effective communication often demands t hat language serve multiple functions.

3. Forms of Discourse

. Sentences are commonly divided into four grammatical forms: a. declarative, b. interrogative, c. imperative, and d. exclamatory.

. Much discourse is intended to serve two or possibly all three functions of lan guage informative, expressive, directive at once. In such cases each aspect or function of a given pa ssage is subject to its own proper criteria.

. Logicians are most concerned with truth and falsehood and the related notions of the correctness and incorrectness of arguments. Thus, to study logic we must be able to differen tiate discourse that functions informatively from discourse that does not.

. argumentation/ persuasion one of the four forms of discourse which uses logic, ethics, and emotional appeals (logos, ethos, pathos) to develop an effective means to convin ce the reader to think or act in a certain way

. persuasion relies more on emotional appeals than on facts

. argument form of persuasion that appeals to reason instead of emotion to convi nce an audience to think or act in a certain way

. description a form of discourse that uses language to create a mood or emotion exposition one of the four major forms of discourse, in which something is explained or "set forth " narrative the form of discourse that tells about a series of events.

4. Emotive Words

. Emotive words are words that carry emotional overtones. These words are said t o have emotive significance or emotive meaning or emotional impact.

1. Two different words or phrases can have literal (or denotative) meanings whic h are similar, but differ significantly in their emotive significance. 2. Often, we speak of "slanting" as emotive significance; i.e., a word or phrase can be positively slanted, neutral, or negatively slanted.

. The informative function derives from the literal meaning of the words in the sentence the objects, events, or attributes they refer to and the relationship among them asserted by th e sentence. The expressive content emerges because some of the words in the sentence may also ha ve emotional suggestiveness or impact. Words, then, can have both a literal meaning and an em otive meaning. The literal meanings and the emotive meanings of a word are largely independent of one another. . Language has a life of its own, independent of the facts it is used to describ e.

. The game confirms what common experience teaches: One and the same thing can b e referred to by words that have very different emotive impacts.

5. Kinds of Agreement and Disagreement

. Aexcessive reliance on emotively charged language can create the appearance of disagreement between parties who do not differ on the facts at all, and it can just as easily disguise substantive disputes under a veneer of emotive agreement. Since the degrees of agreement in belief and attitude are independent of each other, there are four possible combinations at work here: 1. Agreement in Belief and Agreement in Attitude;

2. Agreement in Belief but Disagreement in Attitude; 3. Disagreement in Belief but Agreement in Attitude; 4. Disagreement in Belief and Agreement in Attitude.

. Agreement in belief and agreement in attitude: There aren't any problems in th is instance, since both parties hold the same positions and have the same feelings about them. . Ex. Mr. Recto: The sun is very far since it s 90 million miles away. . Ms. Grace: Yes that is very far, indeed.

. Agreement in belief but disagreement in attitude: This case, if unnoticed, may become the cause of endless (but pointless) shouting between people whose feelings differ sharply ab out some fact upon which they are in total agreement. . Ex.Mr. Abella: The sun is not so far; it s only 93 million miles away. . Ms. Tiffany: The sun is, indeed, very far since it s 93 million miles away.

. Disagreement in belief but agreement in attitude: In this situation, parties m ay never recognize, much less resolve, their fundamental difference of opinion, since they are lulle d by their shared feelings into supposing themselves allied. . Ex. Mr. Magarin: The sun is incredibly far from the earth; it s 60 million miles away. . Ms. Smith: Yes, the sun is extremely far from the earth, but it s 90 million mil es away.

. Disagreement in belief and disagreement in attitude: Here the parties have so little in common that communication between them often breaks down entirely. . Ex. Mr. Cade: The sun is really very close to earth, only 60 million miles. . Ms. Cade: No, the sun is incredibly far away; it s over 93 million miles from ea rth.

. It is often valuable, then, to recognize the levels of agreement or disagreeme nt at work in any exchange of views. That won't always resolve the dispute between two parties, of course, but it will ensure that they don't waste their time on an inappropriate method of argument o r persuasion.

5. Emotively Neutral Language

. Neutral language is to be preferred when factual truth is our objective. When we are trying to learn what really is the case, or trying to follow an argument, distractions will be f rustrating; and emotion is a powerful distraction. Therefore, when we are trying to reason about facts, referring to them in emotive language is a hindrance.

. Language that is altogether neutral may not be available when we deal with som e very controversial matters. Language that is heavily charged with emotional meaning is unlikely to advance the quest for truth.

. If our aim is to communicate information, and if we wish to avoid being misund erstood, we should use language with the least possible emotive impact.

B. Definition

1. Disputes, Verbal Disputes and Definitions 2. Kinds of Definition and the Resolution of Disputes 3. Denotation (Extension) and Connotation (Intension) 4. Extension, and Denotative Definitions 5. Intension, and Connotative Definition 6. Rules for Definition by Genus and Difference

1. Disputes, Verbal Disputes and Definitions

Disputes: Disputes have their origins in disagreements between individuals.

The disagreement only becomes a dispute when one or other party cannot live with the consequences of the disagreement, and insists on having it resolved.

Disputes mostly arise either from a genuine difference of opinion or from dising enuous self-interest.

2. Kinds of Definition and the Resolution of Disputes

Dispute Resolution: Generally refers to one of several different processes used to resolve disputes between parties, including negotiation, mediation, arbitration, collaborative law, and litigation. Dispute resolution is the process of

resolving a dispute or a conflict by meeting at least some of each side s needs an d addressing their interests. Dispute resolution, or conflict resolution to use another common term, is a rela tively new field, emerging after World War II. Scholars from the Program on Negotiation were leaders in est ablishing the field.

. Adjudicative processes, such as litigation or arbitration, in which a judge, j ury or arbitrator determines the outcome. . Consensual processes, such as collaborative law, mediation, conciliation, or n egotiation, in which the parties attempt to reach agreement. . Not all disputes, even those in which skilled intervention occurs, end in reso lution. Such intractable disputes form a special area in dispute resolution studies.

Theoretical definitions: Are special cases of stipulative or precising definition, distinguished by their attempt to establish the use of this term within the context of a broader intellectual framework. Since the adop tion of any theoretical definition commits us to the acceptance of the theory of which it is an integral part, we are rightly cautious in agreeing to it

Persuasive definition: Is an attempt to attach emotive meaning to the use of a term. Since this can onl y serve to confuse the literal meaning of the term, persuasive definitions have no legitimate use.

The most common way of preventing or eliminating differences in the use of langu ages is by agreeing on the definition of our terms. Since these explicit accounts of the meaning of a word or phrase can be offered in distinct contexts and employed in the service of different goals, it's useful to distinguish definitions of several kinds:

Kinds of Definition: 1. Stipulative Definition 2. Lexical Definition

Stipulative Definition: At the other extreme, a stipulative definition freely assigns meaning to a compl etely new term, creating a usage that had never previously existed.

Lexical Definition: A lexical definition simply reports the way in which a term is already used with in a language community.

Kinds of Resolutions of Disputes: 1. 2. 3. 4. Judicial Dispute Resolution Extrajudicial Dispute Resolution Online Dispute Resolution Genuine and Verbal Resolution

Judicial Dispute Resolution:

. The legal system provides a necessary structure for the resolution of many dis putes. However, some disputants will not reach agreement through a collaborative processes. Some disp utes need the

coercive power of the state to enforce a resolution. Perhaps more importantly, m any people want a professional advocate when they become involved in a dispute, particularly if th e dispute involves perceived legal rights, legal wrongdoing, or threat of legal action against them .

. The most common form of judicial dispute resolution is litigation. programs an nexed to the courts, to facilitate settlement of lawsuits.

Extrajudicial Dispute Resolution

. Some use the term dispute resolution to refer only to alternative dispute reso lution (ADR), that is, extrajudicial processes such as arbitration, collaborative law, and mediation us ed to resolve conflict and potential conflict between and among individuals, business entities, governm ental agencies, and (in the public international law context) states.

Online Dispute Resolution

. Dispute resolution can also take place on-line or by using technology in certa in cases. Online dispute resolution, a growing field of dispute resolution, uses new technologies to solv e disputes. Online Dispute Resolution is also called "ODR". Online Dispute Resolution or ODR also i nvolves the application of traditional dispute resolution methods to disputes which arise on line

Genuine and Verbal Resolution

. Genuine disputes involve disagreement about whether or not some specific propo sition is true. Since the people engaged in a genuine dispute agree on the meaning of the words by mea ns of which they convey their respective positions, each of them can propose and assess logical a

rguments that might eventually lead to a resolution of their differences.

. Merely verbal disputes, on the other hand, arise entirely from ambiguities in the language used to express the positions of the disputants. A verbal dispute disappears entirely on ce the people involved arrive at an agreement on the meaning of their terms, since doing so re veals their underlying agreement in belief.

3. Denotation (Extension) and Connotation (Intension)

4. Extension, and Denotative Definitions

A denotative definition tries to identify the extension of the term in question. Thus, we could provide a denotative definition of the phrase "this logic class" simply by listing all of our names.

The extension of a general term is just the collection of individual things to w hich it is correctly applied. Thus, the extension of the word "chair" includes every chair that is (or ever has been or ever will be) in the world.

5. Intension, and Connotative Definition

The intension of a general term, on the other hand, is the set of features which are shared by everything to which it applies. Thus, the intension of the word "chair" is (something like) "a piece of furniture designed to be sat upon by one person at a time."

A connotative definition tries to identify the intension of a term by providing a synonymous linguistic expression or an operational procedure for determining the applicability of the term.

6. Rules for Definition by Genus and Difference

Classical logicians developed an especially effective method of constructing con notative definitions for general terms, by stating their genus and differentia.

Five rules by means of which to evaluate the success of connotative definitions by genus and differentia:

1. Focus on essential features. Although the things to which a term applies may share many distinctive properties, not all of them equally indicate its true nature.

2. Avoid circularity. Since a circular definition uses the term being defined as part of its own definition, it can't provide any useful information; either the audience already understands the meaning of the term, or it cannot understand the explanation that includes that term. Thus, for example, there isn't much point in defining "cordless 'phone" as "a telephone th at has no cord."

3. Capture the correct extension. A good definition will apply to exactly the sa me things as the term being defined, no more and no less. There are several ways to go wrong. Con sider

alternative definitions of "bird": "warm-blooded animal" is too broad, since that would include horses, dogs, and a ardvarks along with birds. "feathered egg-laying animal" is too narrow, since it excludes those birds who h appen to be male, and "small flying animal" is both too broad and too narrow, since it includes bats ( which aren't birds) and excludes ostriches (which are). Successful intensional definitions must be satisfied by all and only those thing s that are included in the extension of the term they define.

4. Avoid figurative or obscure language. Since the point of a definition is to e xplain the meaning of a term to someone who is unfamiliar with its proper application, the use of lang uage that doesn't help such a person learn how to apply the term is pointless. Thus, "happ iness is a warm puppy" may be a lovely thought, but it is a lousy definition.

5. Be affirmative rather than negative. It is always possible in principle to ex plain the application of a term by identifying literally everything to which it does not apply. In a f ew instances, this may be the only way to go: a proper definition of the mathematical term "infinit e" might well be negative, for example. But in ordinary circumstances, a good definition uses pos itive designations whenever it is possible to do so. Defining "honest person" as "some one who rarely lies" is a poor definition.

IV. DEDUCTIVE REASONING

A. Categorical Propositions

1. Categorical Propositions and Classes 2. Quality, Quantity and Distribution 3.The Traditional Square of Opposition 4. Further Immediate Inferences 5 Existential Import 6. Symbolism and Diagrams for Categorical Propositions

1. Categorical Propositions and Classes:

. Propositions are Statements or sentences where the content or meaning of a mea ningful declarative sentence. Statements or sentences that posses a quality or property of being either TRUE or FALSE. External manifestation of the mental product of Judgement

. Categorical Propositions are IS THE KIND OF PROPOSITION WHEREIN THE JUDGEMENT IS DONE IN ABSOLUTE MANNER, i.e., The agreement or disagreement between the subject and the predicate, is done in an absolute manner and MAKES A DIRECT ASSERTION OF AGREEMENT BETWEEN THE SUBJECT AND THE PREDICATE.

. Examples: a) HONEST FILIPINOS AVOID CHEATERS. (asserts that the entire class of honest Filipinos is included in the class of p eople who avoid liars.) b) PEDICABS DO NOT BELONG TO EXPRESSWAYS.

(asserts that the entire class of Pedicabs is excluded from the class of vehicle s that beling to expressways.) c) MANY STUDENTS HAVE CELLPHONES. d) NOT ALL LONG DISTANCE RELATIONSHIPS HAVE HAPPY ENDINGS. e) ANN CURTIS IS A FAMOUS SHOWBIZ PERSONALITY.

Classes of Categorical Propositions: 1. THOSE THAT ASSERT THAT THE WHOLE SUBJECT CLASS IS INCLUDED IN THE PREDICATE CLASS.

2. THOSE THAT ASSERT THAT PART OF THE SUBJECT CLASS IS INCLUDED IN THE PREDICATE CLASS.

3. THOSE THAT ASSERT THAT THE WHOLE SUBJECT CLASS IS EXCLUDED FROM THE PREDICATE CLASS

4. THOSE THAT ASSERT THAT PART OF THE SUBJECT CLASS IS EXCLUDED FROM THE PREDICATE CLASS.

3 Elements of a Proposition: 1. Subject This man.. 2. Coupla is...

3. Predicate

a doctor.

2. Quantity, Quality and Distribution

Quantity of a Proposition: Is equivalent of a quantity of a subject.

1. SINGULAR - Stands for a single definite individual group - Example: Aristotle is the father of logic.

2. PARTICULAR - The subject designates an indefinite part of its total extension. - Example: Some philosophers are atheists.

3. UNIVERSAL - subject can apply to every portion of the term being indicated. - Example: Love is not selfish.

Quantity of a Predicate: 1. Singular - The predicate indicates any signs of singularity - Ex. The new Dean of the College of Law of LDCU is Atty.Adel Tamano.

2. Particular - The predicate of a categorical is not singular and the copula is affirmative. - A truly happy life is a life of goodness

3. Universal - The predicate is not singular, and the copula is negative, then the predicate is universal. - Most politicians are not moral persons.

A Proposition E Proposition

UNIVERSAL / SINGULAR QUANTITY

UNIVERSAL / SINGULAR QUANTITY

AFFIRMATIVE QUALITY NEGATIVE QUALITY PARTICULAR QUANTITY PARTIcULAR QUANTITY AFFIRMATIVE QUALITY NEGATIVE QUALITY

I Proposition O Proposition

A Proposition: EXAMPLES: 1. ALL SOLDIERS ARE PATRIOTIC. 2. EVERY PHILOSOPHER IS A LOVER OF WISDOM. 3. AN ORANGUTAN IS AN APE.

*Quantity is UNIVERSAL/SINGULAR Quality is AFFIRMATIVE

E Proposition: EXAMPLES: 1. NO SINNERS ARE SAINTS. 2. A GREEN MANGO IS NOT SWEET. 3. NEITHER DIAMONDS NOR GOLD IS EXPENSIVE.

*Quantity is UNIVERSAL/SINGULAR Quality is NEGATIVE

I Proposition: EXAMPLES: 1. SOME PHILOSOPHERS ARE ATHEISTS. 2.FILIPINOS ARE NATURE-LOVERS. 3. ALMOST ALL PEOPLE ARE GOD-FEARERS.

* Quantity

PARTICULAR Quality -AFFIRMATIVE

images.jpg O Proposition: EXAMPLES: 1. NOT ALL SENATORS ARE HONEST POLITICIANS. 2.NOT EVERYONE WHO CALLS TO ME LORD, LORD ARE PERSONS TO BE SAVED.

3. SOME FILIPINOS ARE NOT PATRIOTIC.

*Quantity

PARTICULAR Quality - NEGATIVE

Quality of a Proposition: 1. AFFIRMATIVE affirms class membership. - connotes that all, or some, of the members indicated by the subject are contai ned in the class indicated by the predicate.

2. NEGATIVE denies class membership. If all, or some of the members indicated by the subject are not contained in the class indicated by the predicate

3.The Traditional Square of Opposition . . . . CONTRARIES CANNOT BOTH BE TRUE, BUT BOTH CAN BE FALSE. SUBCONTRARIES CANNOT BOTH BE FALSE, BUT BOTH CAN BE TRUE. SUBALTERN PAIRS CAN BOTH BE TRUE OR BOTH BE FALSE. CONTRADICTORIES CANNOT BE TRU AND CANNOT BE FALSE.

4. Further Immediate Inferences 1. Conversion - is the new categorical proposition that results from putting the PREDICATE term of the original proposition in the SUBJECT place of the NEW PROPOSITION and the SUBJECT term of the original in the PREDICATE place of the new.

. Example. No cats are canines. Some snakes are poisonous animals No canines are cats. Some poisonous Animals are snakes

- The converse of any E or I propositions are true only if the original proposit ion was true. From either pair of examples above, both propositions are true or both are false. That is why conversion grounds an immediate inference for both E and I propositi ons.

2. Subalternation - if we first perform a subalternation and then convert our result, then the tru th of an A proposition may be said, in "conversion by limitation," to entail the truth of a n I proposition with subject and predicate terms reversed

. Example: All dogs are mammals" to be true while "All mammals are dogs" is fals e

"Some females are not mothers" to be true while "Some mothers are not females" i s false. Thus, conversion does not warrant a reliable immediate inference with res pect to A and O propositions.

3. Obversion - In order to form the obverse of a categorical proposition, we replace the pred icate term of the proposition with its complement and reverse the quality of the propo sition, either from affirmative to negative or from negative to affirmative

. Example: "All ants are insects" is "No ants are non-insects" "No fish are mammals" is "All fish are non-mammals"

- Obversion is the only immediate inference that is valid for categorical propos itions of every form. In each of the instances cited above, the original proposition and i ts obverse must have exactly the same truth-value, whether it turns out to be true or false .

4. Contrapositions - The contrapositive of any categorical proposition is the new categorical propo sition that results from putting the complement of the predicate term of the original proposition in the subject place of the new proposition and the complement of th e subject term of the original in the predicate place of the new

. Examples: "Some carnivores are not mammals" is "Some non-mammals are not noncarnivores."

- the contrapositive of any A or O proposition is true if and only if the origin al proposition was true. If we form the contrapositive of our result after performing subalter nation, then an E proposition, in "contraposition by limitation," entails the truth of a related O proposition

. If "No bandits are biologists" then "Some non-biologists are not non-bandits,"

(Provided that there is at least 1 member of the class designated by

bandits )

- contraposition is not valid for E and I propositions. . Example: "No birds are plants" and "No non-plants are non-birds" need not have the same truth-value

5. Existential Import . A term, whether subject or predicate, is said to have existential import if the term implies the actual existence of members of that category. Aristotelian logic inferred affirmative propositions such as "All cats are felines" include that there are such entities as cats. Aristotelian logic also inferred the existence of members of I propositions. "Some diesel engines are engines powered by coconut oil," also implied the existence of such diesel engines

6. Symbolism and Diagrams for Categorical Propositions

. The modern interepretation of categorical logic also permits a more convenient way of assessing the truth-conditions of categorical propositions, by drawing Venn diagrams, topological representations of the logical relationships among the classes designated by categorical terms. The basic idea is fairly straightforward:

Each categorical term is represented by a labelled circle. The area inside the circle represents the extension of the categorical term, and the area outside the circle its complement. Thus, members of the class designated by the categorical term would be located within the circle, and everything else in the world would be located outside it.

We indicate that there is at least one member of a specific class by placing an inside the circle; an outside the circle would indicate that there is at least one member of the complementary class.

To show that there are no members of a specific class, we shade the entire area inside the circle; shading everything outside the circle would indicate that there are no members of the complementary class.

In order to represent a categorical proposition, we must draw two overlapping ci rcles, creating four distinct areas corresponding to four kinds of things: 1. those that are members of the class designated by the subject term but not o f that designated by the predicate term; 2. those that are members of both classes; 3. those that are members of the class designated by the predicate term but not of that designated by the subject term; 4. and those that are not members of either class.

The universal negative (E) proposition asserts that nothing is a member of both classes designated by its terms, so its diagram shades the area in which the two circles overlap.

The particular affirmative (I) proposition asserts that there is at least one thing that is a member of both classes, so its diagram places an in the area where the two circles overlap.

two diagrams models the contradictory relationship between E and I propositions; one of them must be true and the other false, since either there is at least one member that the two classes have in common or there are none.

The particular negative (O) proposition asserts that there is at least one thing that is a member of the class designated by its subject term but not of the class designated by its predicate term, so its diagram places an in the area inside the circle that represents the subject term but outside the circle that represents the predicate term.

Finally, the universal affirmative (A) proposition asserts that every member of the subject class is also a member of the predicate class. Since this entails that there is nothing that is a member of the subject class that is not a member of the predicate class, an A proposition can be diagrammed by shading the area inside the subject circle but outside the predicate circle.

B. Categorical Syllogisms

1. Standard-Form Categorical Syllogisms 2. The Formal Nature of Syllogistic Argument 3. Venn Diagram: Technique for Testing Syllogisms 4. Six Rules of Categorical Syllogisms

1. Standard-Form Categorical Syllogism

- A categorical syllogism is a verbal expression of an inference. It is an oral or written discourse showing the agreement or disagreement between two terms on the basis of their re spective relation to a common third term

- Is any argumentation in which, from two prepositions called the premise, we co nclude a third proposition called the conclusion, which is so related to the premise taken join tly that if the premises are true, the conclusion must also be true

- The logical form is the structure of a categorical syllogism indicated by its figures and moods.

. Figure is the arrangement of the terms. major, minor, middle terms of the argument.

a. Major term is found in the major premise, used either as subject or predicate, or as predicate of the conclusion. Since the major term has the greatest extension, it has the greatest concept compared to the other terms.

b. Minor term is the term whose function is to mediate between the other two terms. In an affirmative syllogism. The middle term unites the major term and the minor term. In a negative syllogism, it separates the two. thus, the quality of the syllogism depends on the role of the middle term toward the other two terms. The middle term is never to be found in the conclusion. It is used either as subject or predicate in the premises.

. Mood is the arrangement of the preposition by quantity or quality.

2. The Formal Nature of Syllogistic Argument

A categorical syllogism must always have that SEQUENTIAL RELATION as a differentiating mark of a true and valid syllogism from what is not.

SEQUENTIAL RELATION refers to the interdependence of the premise upon one another. The sure sign of the sequential relation is the presence of a middle term in the premise. NOT A VALID SYLLOGISM. . No sequential relation between the first and second premise because there is no

middle term to connect or disconnect the two terms. . There is no relation between the two premise in which to derive a valid conclusion. . The above supposed conclusion is no conclusion at all.

EXAMPLE: Every man is biped. But every cow is quadruped. Therefore, every cow is not a man.

. VALID AND TRUE SYLLOGISM . There exist a sequential flow of thought from the first premise to the second premise, and then from the premise to the conclusion. . The propositions are connected to one another through the middle term creature which connects the major term mortal and the minor term person.

EXAMPLE: All creatures are mortal. But a person is a creature. Therefore, a person is mortal.

21.jpg minor.jpg 20.jpg

A syllogism is called categorical if the premise and the conclusion composing it are categorical propositions expressed in a declarative form like women are beautiful . not all Filipinos are poor. Or every creature is good .

3. Venn Diagram: Technique for Testing Syllogisms In order to test a categorical syllogism by the method of Venn diagrams, one mus t first represent both of its premises in one diagram. That will require drawing three overlapping circles, fo r the two premises of a standard-form syllogism contain three different terms-minor term, major term, an d middle term. How to draw Venn Diagram:

STEP 1:

First draw three overlapping circles and label them to represent the major, minor, and middle terms of the syllogism.

No M are P. Some M are S. Therefore, Some S are not P.

STEP 2:

Since the major premise is a universal proposition, we may begin with it. The diagram for "No M are P" must shade in the entire area in which the M and P circles overlap. (Notice that we ignore the S circle by

shading on both sides of it.)

22.jpg 23.jpg 24.jpg

Now we add the minor premise to our drawing. The diagram for "Some M are S" puts an inside the area where the M and S circles overlap. But part of that area (the portion also inside the P circle) has already been shaded, so our must be placed in the remaining portion.

Next, on this framework, draw the diagrams of both of the syllogism's premises. a. Always begin with a universal proposition, no matter whether it is the major or the minor premise. b. Remember that in each case you will be using only two of the circles in each case; ignore the third circle by making sure that your drawing (shading or ) straddles it.

STEP 3:

Ignoring the M circle entirely, we need only ask whether the drawing of the conclusion "Some S are not P" has already been drawn.

Finally, without drawing anything else, look for the drawing of the conclusion. If the syllogism is valid, then that drawing will already be done.

Since it perfectly models the relationships between classes that are at work in categorical logic, this procedure always provides a demonstration of the validity or invalidity of any c ategorical syllogism.

Here is a diagram of a syllogistic form. In such case, both of the premises have already been drawn in the appropriate way, so if the drawing of the conclusion is already drawn, the syllogism must be valid, and if it is not, the syllogism must be invalid.

AAA-1 (valid) All M are P. All S are M. Therefore, All S are P.

4. Six Rules of Categorical Syllogisms Rule 1: A syllogism must contain exactly three terms, each of which is used in t he same sense. - The use of exactly three categorical terms is part of the definition of a cate gorical syllogism, and we saw earlier that the use of an ambiguous term in more than one of its senses amounts to the use of two distinct terms. In categorical syllogisms, using more than three terms commits the fallac y of four terms.

This syllogism appears to have only three terms, but there are really four since one of them, the middle term power is used in different senses in the two premises. Example of INVALID SYLLOGISM: Power tends to corrupt. Knowledge is power. Knowledge tends to corrupt

Rule 2: In a valid categorical syllogism the middle term must be distributed in at least one of the premises. - In order to effectively establish the presence of a genuine connection between the major and minor terms, the premises of a syllogism must provide some information about the entire class designated by the middle term. If the middle term were undistributed in both premises, then the two porti ons of the designated class of which they speak might be completely unrelated to each other. Syllogisms that violate this rule are said to commit the fallacy of the undistributed middle. The middle term is what connects the major and the minor term. If the middle term is never distributed, then the major and minor terms might be related to different parts of the M class, thus giving no common ground to relate S and P.

Example of INVALID SYLLOGISM: All sharks are fish All salmon are fish

All salmon are sharks

Rule 3: In a valid categorical syllogism if a term is distributed in the conclus ion, it must be distributed in the premises. - A premise that refers only to some members of the class designated by the majo r or minor term of a syllogism cannot be used to support a conclusion that claims to tell us about ev ery member of that class. Depending which of the terms is misused in this way, syllogisms in violation com mit either the fallacy of the illicit major or the fallacy of the illicit minor. When a term is distributed in the conclusion, let s say that P is distributed, then that term is saying something about every member of the P class. If that same term is NOT distributed in the major premise, then the major premise is saying something about only some members of the P class.

Example of INVALID SYLLOGISM: All horses are animals Some dogs are not horses Some dogs are not animals

Rule 4: No syllogism can have two negative premises. - The purpose of the middle term in an argument is to tie the major and minor te rms together in such a way that an inference can be drawn, but negative propositions state that the terms o f the propositions are exclusive of one another. In an argument consisting of two negative propositions the middle term is excluded from both the major term and the minor term, and thus there is no connection bet ween the two and no inference can be drawn. A violation of this rule is called the fallacy of exclus ive premises.

If the premises are both negative, then the relationship between S and P is denied. The conclusion cannot, therefore, say anything in a positive fashion. That information goes beyond what is contained in the premises.

Example of INVALID SYLLOGISM: No fish are mammals Some dogs are not fish Some dogs are not mammals

Rule 5: If either premise of a valid categorical syllogism is negative, the conc lusion must be negative. - An affirmative proposition asserts that one class is included in some way in a nother class, but a negative proposition that asserts exclusion cannot imply anything about inclusion. For th is reason an argument with a negative proposition cannot have an affirmative conclusion. An argument that vio lates this rule is said to commit the fallacy of drawing an affirmative conclusion from a negative premise. Example of INVALID SYLLOGISM: All crows are birds Some wolves are not crows Some wolves are birds Rule 6. In valid categorical syllogisms particular propositions cannot be drawn properly from universal premises. - Because we do not assume the existential import of universal propositions, the y cannot be used as premises to establish the existential import that is part of any particular proposition. The existential fallacy violates this rule. Although it is possible to identify additional features shared by all vali d categorical syllogisms (none of them, for example, have two particular premises), these six rules are jointly su fficient to distinguish between valid and invalid syllogisms.

Thus, if the syllogism has universal premises, they necessarily say nothing about existence. Yet if the conclusion is particular, then it does say something about existence. In which case, the conclusion contains more information than the premises do, thereby making it invalid. Example of INVALID SYLLOGISM: All mammals are animals All tigers are mammals Some tigers are animals

C. Arguments in Ordinary Language

1. Reducing the Number of Terms in a Syllogistic Argument 2. Translating Categorical Propositions into Standard Form 3.Uniform Translation 4.Enthymemes 5 Sorites 6. Disjunctive and Hypothetical Syllogisms 7.The Dilemma

1. Reducing the Number of Terms in a Syllogistic Argument In slightly more complicated instances, an ordinary argument may deal with more than three terms, but it may still be possible to restate it as a categorical syllogism. Two kinds of too ls will be helpful in making such a transformation: STEP 1: First, it is always legitimate to replace one expression with another th at means the same thing. Of course, we need to be perfectly certain in each case that the expressions are ge nuinely synonymous. But in many contexts, this is possible: in ordinary language, "husbands" and "married m ales" almost always mean the same thing. STEP 2: Second, if two of the terms of the argument are complementary, then appr opriate application of the immediate inferences to one of the propositions in which they occur will enable us to reduce the two to a single term. Consider, for example, "No dogs are non-mammals, and some non-canines are not n on-pets, so some nonmammals are pets." Replacing the first proposition with its (logically equivalen t) obverse, substituting "dogs" for the synonymous "canines" and taking the contrapositive of the second, and ap plying first conversion and then obversion to the conclusion, we get the equivalent standard-form categorica l syllogism: All dogs are mammals. The invalidity of this syllogism is more readily apparent than that of the argument from which it was derived. Some pets are not dogs.

Therefore, Some pets are not mammals.

2. Translating Categorical Propositions into Standard Form We may need only to re-arrange the propositions of the argument in order to tran slate it into a standard-form categorical syllogism.

Thus, for example, "Some birds are geese, so some birds are not felines, since n o geese are felines" is just a categorical syllogism stated in the non-standard order minor premise, conclusion, major premise; all we need to do is put the propositions in the righ t order, and we have the standard-form syllogism: No geese are felines. Some birds are geese. Therefore, Some birds are not felines.

3.Uniform Translation

In order to achieve the uniform translation of all three propositions contained in a categorical syllogism, it is sometimes useful to modify each of the terms employed in an ordinary-language ar gument by stating it in terms of a general domain or parameter. The goal here, as always, is faithfully to represent the intended meaning of eac h of the offered propositions, while at the same time bringing it into conformity with the others, making it po ssible to restate the whole as a standard-form syllogism. Thus, for example, in the argument, "The attic must be on fire, since it's full of smoke, and where there's smoke, there's fire," the crucial parameter is location or place. If we suppose the terms of this argument to be "places where fire is," "places where smoke is," and "places that are the att ic," then by applying our other techniques of restatement and re-arrangement, we can arrive at the syllogism: This standard-form categorical syllogism of the form AAA-1 is clearly valid.

All places where smoke is are places where fire is. All places that are the attic are places where smoke is. Therefore, All places that are the attic are places where fire is.

4.Enthymemes - Another special case occurs when one or more of the propositions in a categori cal syllogism is left unstated. Incomplete arguments of this sort, called enthymemes are said to be "first-," "s econd-," or "third-order," depending upon whether they are missing their major premise, minor premise, or c onclusion respectively. In order to show that an enthymeme corresponds to a valid categorical syllogism, we need only supply the missing premise in each case. Thus, for example, "Since some hawks have sharp beaks, some birds have sharp bea ks" is a second-order enthymeme, and once a plausible substitute is provided for its missing minor pre mise ("All hawks are birds"), it will become the valid IAI-3 syllogism: Some hawks are sharp-beaked animals. All hawks are birds. Therefore, Some birds are sharp-beaked animals.

5.Sorites Finally, the pattern of ordinary-language argumentation known as sorites involve s several categorical syllogisms linked together. The conclusion of one syllogism serves as one of the premises for another syllogism, whose conclusion may serve as one of the premises for another, and so on. In any such case, of course, the whole procedure will comprise a valid inference so long as each of t he connected syllogisms is itself valid.

6. Disjunctive and Hypothetical Syllogisms (KINDS OF COMPOUND PROPOSITION) DISJUNCTIVE:

The disjunctive holds that at least one of the two components are true, allowing for the possibility that both are true. If we have a disjunction as one premise, and a denial of one of the di sjuncts as a second premise, we can validly infer that the other disjunct component is true P1 Either that's a gun in your pocket, or you're happy to see me P2 You don't have a gun in your pocket (This is implied - implied premises are c alled "enthymemes") C: You must be happy to see me

This argument is valid, because it eliminates one of the disjuncts. Take a look at this argument: P1 Either that's a gun in your pocket, or you're happy to see me P2 You have a gun in your pocket C: Therefore you're not happy to see me

This argument is invalid. It bears a superficial similiarty to the above argumen t, but rather than eliminate one of the disjuncts, it merely affirms one of them. HYPOTHETICAL SYLLOGISMS: We can call these statements If/Then statements, where the "If" part is the ante cedant and the part following after "Then" is the consequent. A conditional that contains conditional statements exclusively is called a pure hypothetical syllogism: Example: P1: If you study (antecedent), then you will become a good student (con sequent). P2: If you become a good student, then you will go to college Therefore, If you study, then you will go to college.

Notice that the first premise and the conclusion have the same antecedent, and t he second premise and the conclusion have the same consequent.

There are two valid and two invalid forms of a mixed hypothetical syllogism: a) modus ponens (From the Latin "ponere", "to affirm") b) modus tollens (Latin: "To deny")

The next form, Affirming the consequent, is invalid: Affirming the consequent . If P is true then Q is true . Q is true . Therefore, P is true

MODUS PONENS: If P is true, then Q is true P is true Therefore, Q is true

MODUS TOLLENS: If P is true, then Q is true Here the syllogism denies the consequent of the conditional premise, and the conclusion denies the antecedant.

Q is not true Therefore, P is not true

7.The Dilemma It is a form of argument that is composed of a conjunction of two conditional h ypothetical statements as its major premise. KINDS: 1. Simple Dilemma . A form wherein the conclusion is a categorical proposition. Example: The security officer of a certain university was called into the office of the c hief security on the issue of allowing students to enter the school premises without wearing their ID s. The chief security said to the guard: Either you were playing favorites or you were not; if you were playing favorites, then you should be punished for being incompetent. If you were not playing favorites, you deserve p unishment for dereliction of duty. Therefore, in either case, you should be punished.

2. Complex Dilemma . It is a form of dilemma wherein the conlusion is a disjunctive proposition off ering alternatives. Example: If the next president of the Phil. Is a re-electionist, then he has nothing new to offer. If the next president of the Phil. Is not a re-electionist, then he is not yet equipped in t he country s political and economic matters. Therefore, in either case, the next president of the Philippin es has nothing new to offer or he is not yet equipped in the country s political and economic matters.

D. Symbolic Logic

1. The Value of Special Symbols 2. The Symbols for Conjunction, Negation, and Disjunction 3. Conditional Statements and Material Implication 4. Argument Forms and Arguments 5. Statement Forms, Material Equivalence, Logical Equivalence 6. The Paradoxes of Material Implication 7. The Three Laws of Thought

2. The Symbols for Conjunction, Negation, and Disjunction

Conjunction: In logic, a conjunction is a compound sentence formed by using the word and to j oin two simple sentences. The symbol for this is .. (whenever you see . read 'and') When two si mple sentences, p and q, are joined in a conjunction statement, the conjunction is expressed symbo llically as p . q.

Simple Sentences Compound Sentence : conjunction p: Joe eats fries. q: Maria drinks soda. p . q : Joe eats fries, and maria drinks soda.

Conjunction =

and

Conjunction is only true if both conjuncts are true

Negation:

Indicates the opposite, usually employing the word not. The symbol to indicate n egation is : ~ Original Statement Negation of statment Today is monday. Today is not monday. That was fun. That was not fun.

Negation = not

= ~

Negation of a statement is true if statement is false Negation of a statement is false if statement is true

Disjunction: . In logic, a disjunction is a compound sentence formed by using the word or to join two simple sentences. The symbol for this is .. (whenever you see . read 'or') When two simple sentences, p and q, are joined in a disjunction statement, the disjunction is ex pressed symbollically as p . q.

Pneumonic: the way to remember the symbol for disjuntion is that, this symbol . looks like the 'r' in or, the keyword of disjunction statements. Simple Sentences Compound Sentence : disjunction p: The clock is slow. q: The time is correct. p . q : The clock is slow, or the time is correct.

Warning and caveat: The only way for a disjunction to be a false statement is if BOTH halves are false. A disjunction is true if either statement is true or if both statements a re true! In other words, the statement 'The clock is slow or the time is correct' is a false statement on ly if both parts are false! Likewise, the statement 'Mr. G teaches Math or Mr. G teaches Science' is true if Mr. G is teaches science classes as well as math classes!

Disjunction =

or

= v

Disjunction is true if either disjunct is true

Examples:

1. It is not true that evil spirits exist.

First step: Make a dictionary (define statements) Second step: Look at the sentence, symbolize statements correctly (using v) (Third step: Determine truth values) , ~, or

Answer: It is not true that evil spirits exist. ~E E=Evil spirits exist. If evil spirits do exist (E is True), then ~E is false. If evil spirits do not exist (E is False), then ~E is true.

2. Determine whether the following is true: ~(A v C) v ~(X ~Y) Given: A, B, and C are True X, Y, and Z are False

~(A v C) v ~(X . . . .

~Y)

The main connective = the middle wedge (v) (disjunction) Therefore we have two disjuncts Left disjunct= ~(A v C) Right disjunct = ~(X ~Y)

. Strategy: determine truth values of each disjunct, then we know if at least on e disjunct is true, this will make the whole statement true . ~(A v C) v ~(X ~Y) . Left disjunct: ~(A v C) . Both A and C are true. This makes (A v C) true. . But (A v C) is negated, so ~(A v C) is false. . Right disjunct: ~(X ~Y) . X is false. . Y is false, so this means ~Y is true. . This makes the inner conjunction false (to be true, both conjuncts (X and ~Y) must both be true) . Because the whole statement (X ~Y) is false, this makes its negated form ~(X Y) true . Since the left disjunct is false, and the right disjunct is true, this means ~ (A v C) v ~(X ~Y) is true (since at least one disjunct is true)

3. Conditional Statements and Material Implication

Conditional statement: when two statements are combined by placing the word ore the first and then before the second . If then . We use the arrow . or horseshoe to represent the if-then phrase . Also called a hypothetical, an implication, or an implicative statement

if bef

The component statement that follows the if is called the antecedent The component statement that follows the then is called the consequent If (antecedent), then (consequent) A conditional statement asserts that if its antecedent is true, then its consequ ent is also true But as in disjunction, there are a few different senses in which a conditional c an be interpreted

4 Types of Implications: Logical Implication: the consequent follows logically from its antecedent Example: If all humans are mortal and Socrates is a human, then Socrates is mort al. Definitional Implication: the consequent follows the antecedent by definition Example: If Leslie is a bachelor, then Leslie is unmarried. Causal Implication: The connection between antecedent and consequent is discover ed empirically Example: If I put X in acid, then X will turn red. Decisional Implication: no logical connection nor one by definition between the consequent and antecedent. This is a decision of the speaker to behave in the specified way under the speci fied circumstances Example: Is we lose the game, then I ll eat my hat. Understanding the Implication: No matter what type of implication is asserted by a conditional statement, part of its meaning is the negation of the conjunction of its antecedent with the negation of its consequent For a conditional to be true (e.g. If p then q ), ~(p ~q) must be true:

Think p= A piece of blue litmus paper is placed in that solution. q= The piece of blue litmus paper will turn red. If p then q = false if paper is placed in solution, but doesn t turn red th

The horseshoe symbol does not stand, therefore, for all the meanings of if-then ere are several meanings. p q abbreviates ~(p ~q), whose meaning is included in the meanings of each kind of implication

Material Implication: . represents the material implication. A fifth type of implication E.g. If Hitler was a military genius, then I m a monkey s uncle.

No real connection between antecedent and consequent . This kind of relationship is what is meant by material implication . It just asserts that it is not the case that the antecedent is true when the consequent is false.

. Many arguments contain conditional statements of various kinds of implication, but the validity of all valid arguments (of the general type with which we will be concerned) is preserv ed, even if the additional meanings of their conditional statements are ignored.

If . . . .

can be replaced by such phrases as: in case provided that given that on condition that

Some indicator words for . implies... . entails

then

include:

4. Argument Forms and Arguments

Substitution-Instances Since the statements of the propositional calculus are propositions, they can be combined to form logicalarguments, complete with one or more premises and a single conclusion tha t may follow validly from them. Thus, for example, each of the following is an argument expressed in the l anguage of symbolic logic: A . B (D B) . ~E (A . E) . (D = B)

A D

B A . E

_______ _______________ ________________________

B ~E D = B What is more, notice that all three of these arguments share a common structure: the first premise of each is a . statement; the second premise is the antecedent of that statement; and the c onclusion is its consequent. We can exhibit this common structure more clearly by using statement variables t o express the argument forminvolved: p . q

p ________

q Each of the three arguments above is a substitution instance of this argument fo rm, since each of them results from the substitution of an appropriate (simple or compound) statement f or each of the statement variables in the argument form. Notice that these substitutions must be consiste nt in each application; once we've put D B in the place of p in the first premise of the second argument, for example, we must also put it in the place of p in the second premise. In the same way, the first and third arguments above along with indefinitely many others can be shown to be substitution-instances of the same arg ument form. Most arguments are substitution-instances of several distinct argument forms, each of which can be no more complex in structure than the argument itself. Testing for Validity

Recognizing individual arguments as substitution-instances of more general argum ent forms is an important skill because, as we've already seen, the validity of any argument dep ends solely upon its logical form. An argument in the propositional calculus is valid whenever it is a substi tution-instance of an argument form in which it is impossible for the premises to be true and the conclusion fa lse. Since the argument form reliably leads from premises of a certain general structure to a conclusion of a different structure, every substitution-instance of that argument form must express a valid argument. Thus, the same truth-tables we used to define the statement connectives provide an effective decision procedure for determining the validity of arguments in the propositional calculu s. We simply chart the truthvalues of each premise and the conclusion of an argument form for every possible combination of truthvalues for the statement variables involved, and look to see what happens on tho se lines of the truth-table in which all of the premises are true. If the conclusion is also true on each of th ese lines, then the inference captured by the argument form is a valid one, and arguments of this form must al l be valid. If, however, there is even a single line on which all of the premises are true but the conclusion i s false, then the inference is

invalid, and we cannot be sure whether arguments of this form are valid or inval id. (They certainly are not valid because of this form, but of course some of them may happen to be substitu tion-instances of other argument forms whose inferences are valid.) Modus Ponens Consider, for example, what happens when we construct a truth-table that lists e ach of the four combinations of truth-values that the component statements could exhibit in the simple argument form that we identified at the top of this page.

1st Premise 2nd Premise Conclusion p q p . q p q T T T T T T F F T F F T T

F T F F T F F

p . q

p _______

q This truth-table shows that (no matter what statements we substitute for p and q ) both of the premises of the argument will be true only on the first line (when both component statements are true). But on that line, the conclusion is also true, so the inference is valid. Whenever we come across an argument that shares this basic structure, we can be perfectly certain of its logical validity. In fact, arguments of this form are so common that the form itself has a name, Modus Ponens, which we will usually a bbreviate as M.P. On the other hand, consider what happens when we construct a truth-table for tes ting the validity of a distinct, though superficially similar, argument form:

1st Premise 2nd Premise Conclusion p q p . q

q p T T T T T T F F F T F T T T F F F T F F

p . q

q _______

p In arguments of this form, both premises are true on the first and on the third lines of the truth-table. While the conclusion is

true on the first line, on the third line it is false. Since it is therefore possible for the premises to be true while the conclusion is false, the inference is invalid. This unreliable argument form is called the fallacy of affirming the consequent. Although it might be mistaken for M.P. at a casual glance, the fallacy unlike its valid cousin does not guarantee the truth of its conclusion.

Modus Tollens Another common argument form with a valid inference is Modus Tollens (abbreviate d as M.T.), which has the form:

1st Premise 2nd Premise Conclusion p q p . q ~ q ~ p T T T F F T F F T F F T T F T F F

T T T

p . q

~ q _______

~ p As the truth-table shows, the premises are true only when both of the component statements are false, in which case the conclusion is also true. There is no line on which both premises are true and the conclusion false, so the inference is valid, as are all substitution-instances of this argument form. As with M.P., there is an argument form superficially similar to M.T. that yields entirely different results.

1st Premise 2nd Premise Conclusion p q p . q ~ p ~ q T T T F F

T F F F T F T T T F F F T T T

p . q

~ p _______

~ q This is the fallacy of denying the antecedent. As the truth-table to the right clearly shows, it is an unreliable inference, since it is possible (on the third line) for both of its premises to be true while its conclusion is false. Substitution-instances of this argument form may not be valid. Hypothetical Syllogism

1st Premise 2nd Premise Conclusion p q r p . q q . r p . r T T T T T T T T F T F F

T F T F T T T F F F T F F T T T T T F T F T F T F F T T T T

F F F T T T

A larger truth-table is required to demonstrate the validity of the argument form called Hypothetical Syllogism (H.S.), since it involves three statement variables instead of two, and we must consider all eight of the possible combinations of their truth-values: p . q

q . r _______

p . r Despite its greater size, this truth-table establishes validity in exactly the same way as its more compact predecessors: both premises are true only on the first, fifth, seventh, and eighth lines, and the conclusion is also true on each of these lines. It follows that all arguments sharing in thisgeneral form m ust be valid. Disjunctive Syllogism Finally, consider the argument form known as Disjunctive Syllogism or D.S.

1st Premise 2nd Premise Conclusion p q

p . q ~ p q T T T F T T F T F F F T T T T F F F T F

p . q

~ p _____

The truth-table demonstration of its validity should look familiar by now. Whenever the premises are true (on the third line of the truth table), so is the conclusion. Once again, however, there is a similar form that embodies an invalid inference, the fallacy of affirming the alternative:

1st Premise 2nd Premise Conclusion p q p . q p ~ q T T T T F

T F T T T F T T F F F F F F T

p . q

p _____

~ q In this case, the first line of the truth-table shows that (with our inclusive sense of the . ) it is possible for the premises to be true and the co nclusion false. . 5. Statement Forms, Material Equivalence, Logical Equivalence Statement Forms: Statement Forms In exactly the same sense that individual arguments may be substitution-instance s of general argument forms, individual compound statements can be substitution-instances of general s tatement forms. In

addition, just as we employ truth-tables to test the validity of those arguments , we can use truth-tables to exhibit interesting logical features of some statement forms. Assessing Statement Forms Because all five of our statement connectives are truth-functional, the status o f every statement-form is determined by its internal structure. In order to determine whether a statement form is tautologous, selfcontradictory, or contingent, we simply construct a truth-table and inspect the appropriate column. Consider, for example, the statement form: p q (p . ~q) . ~(p T T T F F T F T T T F T F T T F F T q)

T T

(p . ~q) . ~(p

q)

Since the truth-table shows that statements of this form can be either true or false, depending upon the truth-values of their components, the statement form is contingent.

Tautology A statement form whose column in a truth-table contains nothing but Ts is said t o be tautologous. Consider, for example, the statement form: p ~ p p . ~ p

p \equiv q p \Leftrightarrow q T F T F T T

p . ~p Notice that whether the component statement p is true or false makes no differen ce to the truth-value of the statement form; it yields a true statement in either case . But it follows that any compound statement which is a substitution-instance of this for m no matter what its content can be used only to make true assertions. Contradiction A statement form whose column contains nothing but Fs, on the other hand, is sai d to be selfcontradictory. For example: p ~ p p T F F F T F ~ p

~p

Again, the truth-value of the component statement doesn't matter; the result is always false. Compound statements that are substitution-instances of this statement for m can

never be used to make true assertions. Contingency Of course, most statement forms are neither tautologous nor self-contradictory; their truth-tables contain both Ts and Fs. Thus: p q p . ~ q T T F T F T F T T F F T

p . ~q Since the column underneath it in the truth-table has at least one T and at leas t one F, this statement form is contingent. Statements that are substitution-instances of this statement form may be either true or false, depending upon the truth-value of their compon ent statements. Logical Equivalence: In logic, statements p and q are logically equivalent if they have the same logi cal content. This is a semantic concept; two statements are equivalent if they have the same truth va lue in everymodel (Mendelson 1979:56). The logical equivalence of p and q is sometimes expressed as , Epq, or . However, these symbols are also used for material equivalence; the pro

per interpretation depends on the context. Logical equivalence is different from material equivalen ce, although the two concepts are closely related. Example: The following statements are logically equivalent:

f \rightarrow e \neg e \rightarrow \neg f 1. If Lisa is in France, then she is in Europe. (In symbols, .) 2. If Lisa is not in Europe, then she is not in France. (In symbols, .) Syntactically, (1) and (2) are derivable from each other via the rules of contra position and double negation. Semantically, (1) and (2) are true in exactly the same models (interpretations, valuations); namely, those in which either Lisa is in France is false or Lisa is in Europe is true. (Note that in this example classical logic is assumed. Some non-classical logics do not deem (1) and (2) logically equivalent.) Difference between Material Equivalence: Logical equivalence is different from material equivalence. The material equival ence of p and q (often written p.q) is itself another statement, call it r, in same object language as p and q. rexpresses the idea "p if and only if q". In particular, the truth value of p.q can change from one model to another. The claim that two formulas are logically equivalent is a statement in the metal anguage, expressing a relationship between two statements p and q. The claim that p and q are semantic ally equivalent does not depend on any particular model; it says that in every possible model, p will hav e the same truth value as q. The claim that p and q are syntactically equivalent does not depend on models at all; it states that there is a deduction of q from p and a deduction of p from q. There is a close relationship between material equivalence and logical equivalen ce. Formulas p and q are syntactically equivalent if and only if p.q is a theorem, while p and q are sema ntically equivalent if and only if p.q is true in every model (that is, p.q is logically valid). Material Equivalence: Material Equivalence In similar fashion, Material Equivalence (Equiv.) provides alternative definitions of the = connective. p q p=q = (p.q) (q.p) T

T T T T T F F T F F T F T F F F T T T

Its first form defines = in terms of . , justifying the use of the term "biconditional:" *p=q+=*(p.q) (q.p)]

p q p=q

= (p q).(~p ~q) T T T T T

(\neg p \land p) \to q p \to (q \to p) \neg p \to (p \to q) p \to (q \lor \neg q) (p \to q) \lor (q \to r) \neg(p \to q) \to (p \land \neg q) T F F T F F T F T F F F T T T

Its second form defines = by pointing out its basic truth-conditions: *p=q+=*(p q).(~p ~q)+ Again, the logical equivalence of these three expressions provides us with a convenient way to comprehend and employ what is asserted in any statement of mat erial equivalence. 6. The Paradoxes of Material Implication

The paradoxes of material implication are a group of formulas which are truths o f classical logic, but which are intuitively problematic. One of these paradoxes is the paradox of entailment . The root of the paradoxes lies in a mismatch between the interpretation of the v alidity of

logical implication in natural language, and its formal interpretation in classi cal logic, dating back to George Boole's algebraic logic. In classical logic, implication describes conditional i f-then statements using a truthfunctional interpretation, i.e. "p implies q" is defined to be "it is not the ca se that p is true and q false". Also, "p implies q" is equivalent to "p is false or q is true". For example, "if it is raining, then I will bring an umbrella", is equivalent to "it is not raining, or I will bring an umbrella, or both". This truth-functional interpretation of implication is called material implication or material conditi onal. The paradoxes are logical statements which are true but whose truth is intuitive ly surprising to people who are not familiar with them. If the terms 'p', 'q' and 'r' stand for arbitrary pr opositions then the main paradoxes are given formally as follows: 1. , p and its negation imply q. This is the paradox of entailment. 2. , if p is true then it is implied by every q. 3. , if p is false then it implies every q. This is referred to as 'explosion'. 4. , either q or its negation is true, so their disjunction is implied by every p. 5. , if p, q and r are three arbitrary propositions, then either p implies q or q implies r. This is because if q is true then p implies it, and if it is false th en q implies any other statement. Since r can be p, it follows that given two arbitrary propositions, o ne must imply the other, even if they are mutually contradictory. For instance, "Nadia is in Barce lona implies Nadia is in Madrid or Nadia is in Madrid implies Nadia is in Barcelona." This truism sounds like nonsense in ordinary discourse. 6. , if p does not imply q then p is true and q is false. NB if p were false then it would imply q, so p is true. If q were also true then p would imply q, h ence q is false. This paradox is particularly surprising because it tells us that if one proposition d oes not imply another then the first is true and the second false.

The paradoxes of material implication arise because of the truth-functional defi nition of material implication, which is said to be true merely because the antecedent is false or the consequen t is true. By this criterion, "If the moon is made of green cheese, then the world is coming to an end," is true m erely because the moon isn't made of green cheese. By extension, any contradiction implies anything wha tsoever, since a contradiction is never true. (All paraconsistent logics must, by definition, rej ect (1) as false.) Also, any tautology is implied by anything whatsoever, since a tautology is always tru e. To sum up, although it is deceptively similar to what we mean by "logically foll ows" in ordinary usage, material implication does not capture the meaning of "if... then".

7. The Three

Laws of Thought

The laws of thought are fundamental axiomatic rules upon which rational discours e itself is based. The rules have a long tradition in the history of philosophy and logic. They are laws that guide and underlie everyone's thinking, thoughts, expressions, discussions, etc. 1. Law of identity - If any statement is true, then it is true. The three classic laws of thought are attributed to Aristotle and were foundatio nal in scholastic logic. They are: The law of identity states that an object is the same as itself: A = A. For the law of identity, Aristotle [1] wrote: Now "why a thing is itself" is a meaningless inquiry (for to give meaning to the q uestion 'why' the fact or the existence of the thing must already be evident e.g., that the moon is eclipsed b ut the fact that a thing is itself is the single reason and the single cause to be given in answer to all such questions as why the man is man, or the musician musical, unless one were to answer, 'because each thing is inseparable from itself, and its being one just meant this.' This, however, is common to all things and is a short and easy way with the question.) - Metaphysics, Book VII, Part 17 2. The Law of Non-Contradiction - no statement can be both true and false In logic, the law of non-contradiction ... states, in the words of Aristotle, th at "one cannot say of something that it is and that it is not in the same respect a nd at the same time". [2] (note Aristotle's use of indices: 'respect' and 'time') see the Principle of contradiction 3. The Law of the Excluded Middle - any statement is either true or false

Aristotle wrote that ambiguity can arise from the use of ambiguous names, but ca nnot exist in the facts themselves: It is impossible, then, that 'being a man' should mean precisely not being a man , if 'man' not only signifies something about one subject but also has one significance. And it will not be po ssible to be and not to be the same thing, except in virtue of an ambiguity, just as if one whom we call 'm an', and others were to call 'not-man'; but the point in question is not this, whether the same thing can at the same time be and not be a man in name, but whether it can be in fact

E. The Method of Deduction

1. Formal Proof of Validity 2. The Rule of Replacement 3. Proof of Invalidity 4. Inconsistency

Formal Proof of Validity Consider the following argument: A B B C ~ C A v D \ D To establish the validity of this argument with a truth table would require a ta ble of 16 rows. But we can establish its validity more efficiently by a sequence of elementary arguments, e ach of which is known to be valid. Such a step-by-step procedure is called a formal proof. In general, we can define a formal proof, that a given argument is valid as a se quence of statements each of which is either a premiss of that argument or follows from preceding statements of the sequence by an elementary valid argument, such that the last statement in the sequence is the c onclusion of the argument whose validity is being proved. An elementary valid argument is any argument that is a substitution instance of an elementary valid argument form. It is important to understand that any substitution instance of a n elementary valid argument form is an elementary valid argument. Example: A formal proof of the argument above would look like this: 1. A B

2. B C 1. ~ C 2. A v D

\ D 3. A C1,2, H.S.

6. ~ A 5,3,M.T. 7. D 4,6, D.S. The first four numbered propositions are the premisses of the original argument, followed by its conclusion. Note that the statement of the conclusion here is not a part of the formal proof itself but an informal reminder of the goal of the proof. It also serves to separate the premisses from the rest of the proof. Each of the following numbered steps is a valid conclusion that follows from one or more of the preceding numbered premisses and steps by an elementary valid argument. The notation on the right c onstitutes the justification for the step. Thus step 5, "A C," is a valid conclusion from premisses 1 and 2 b y an elementary valid argument that is a substitution instance of the form called the hypothetical syl logism (abbreviated H.S.). Step 6, "~A," is a valid conclusion from premiss 5 and step 3 by an elementary v alid argument that is a substitution instance of the form calledmodus ponens (M.P.). And finally, step 7 , "D," which is the conclusion

of the original argument, is a valid conclusion from premiss 4 and step 6 by an elementary valid argument that is a substitution instance of the form called the disjunctive syllogism. St ep 7, in other words, shows that "D" follows from the original premisses, and that the argument is valid. Summary Table Rules of Inference: Elementary Valid Argument Forms Name Abbreviation Form Modus Ponens M.P. p q p \ q Modus Tollens M.T. p q ~q \ ~p Hypothetical Syllogism H.S. p q q r \ p r Disjunctive Syllogism D.S. p v q ~p \ q Constructive Dilemma C.D.

(p q) (r s) p v r \ q v s Absorption Abs. p q \ p (p q) Simplification Simp. p q \ p Conjunction Conj. p q

\ p q Addition Add. p \ p v q

The Rule of Replacement There are many valid truth-functional arguments whose validity cannot be proved using only the nine rules of inference given thus far. We may accept, therefore, as an additional principle o f inference the rule of replacement, which permits us to infer from any statement the result of replacin g any component of that statement by any other statement logically equivalent to the component replaced. For example, the principle of double negation (D.N.) asserts that p is logically equivalent to ~ ~ p. This principle allows us to infer from the statement ~ ~ (A v B) that the statement A v B has the same truth value. And from A v B we can deduce A v ~ ~ B.

Rules of Inference: Logically Equivalent Expressions Name Abbreviation Form De Morgan s Theorems De M. ~(p q) [T] (~p v ~q) ~(p v q) [T] (~p ~q) Commutation Com. (p v q) [T] (q v p) (p q) [T] (q p) Association Assoc.

[p v (q v r)] [T] [(p v q) v r] [p (q r)] [T] [(p q) r] Distribution Dist. [p (q v r)] [T] [(p q) v (p r)] [p v (q r)] [T] [(p v q) (p v r)] Double negation D.N. p [T] ~ ~p Transposition Trans. (p q) [T] (~q ~p) Material implication Impl. (p q) [T] (~p v q) Material equivalence Equiv. (p q) [T] (p q) (q p) (p q) [T] [(p q) v (~p ~q)] Exportation Exp. [(p q) r] [T] [(p (q r)]

Tautology Taut. p [T] (p v p) p [T] (p p)

Keep in mind that the replacement of statements by logically equivalent alternat ives is different from the substitution of statements for statement variables. In moving from a stateme nt form to a substitution instance of it or from an argument form to a substitution instance of it, we can substitute any statement for any statement variable, provided that if a statement is substituted for one occu rrence of a statement variable it must be substituted for every other occurrence of that statement variable. Bu t we can replace a statement with a logically equivalent alternative without having to replace any other occu rrence of it. We do not change the truth value of a larger expression if we replace a part of it with a logical ly equivalent alternative. In addition, the nine rules of inference by the substitution of arguments for va lid argument forms apply only to whole lines of a proof. The statement A can be inferred from A B by simplific ation only if A B constitutes a whole line, not if it is part of a larger expression. In contrast, an expressi on can be replaced by a logically equivalent alternative P for ~ ~P, for example wherever it occurs. Although a formal proof of validity is effective in the sense that it can be mec hanically decided of any given sequence whether it is a proof, constructing a formal proof is not an effective procedure. In this respect formal proofs differ from truth tables. The making of truth tables is completely mechanical: Given any truth functional argument we can always construct a truth table to test its validity b y following simple rules of procedure. But we have no effective or mechanical rules for the construction of formal proofs. Here we must think or "figure out" where to begin and how to proceed. Proof of Invalidity For an invalid argument there is, of course, no formal proof of validity. But if we fail to discover a formal proof of validity for an argument, this failure does not prove the argument to b e invalid or that no such proof can be constructed. It may only mean that we have not tried hard enough. A truth table can conclusively prove an argument invalid if it has at least one row with Ts under all the premisses and an F under the conclusion. But as already noted, a truth table for an argument form with many

variables will be cumbersomely large. We can, however, prove an argument invalid without constructing a complete truth table if we can assign values to its component propositions that make the conclusion false a nd the premisses true. This is equivalent to constructing one row of the argument s truth table. And because a n argument is proved invalid by displaying at least one row of its truth table in which all its premi sses are true and its conclusion false, it suffices as a proof of invalidity.

There is no mechanical method of producing the right assignments of truth values to show that the conclusion can be false and all premisses true. A certain amount of trial and er ror is often inevitable. Even so, this method of proving invalidity is almost always shorter and easier than writi ng out a complete truth table.

Inconsistency Example I will pass the class and I will not pass the class. Therefore I am the greatest student who has ever lived. This argument is certainly silly, but when we consider its formal structure we f ind that it is, in fact, valid. The argument can be symbolized as follows: P ~P \ G The premiss is inconsistent. Because P can only be true or false, the conjunctio n P ~P is selfcontradictory and must be false. This means that it is impossible to find a trut h value assignment that makes the premiss of this argument true and the conclusion false at the same time. The argument is therefore valid, which a formal proof confirms. Note that although this argument, and any argument with inconsistent premisses, is valid, it cannot possibly be sound because its premisses must always be false. No conclusion can be establ ished to be true by an argument with inconsistent premisses, because its premisses cannot possibly all be true themselves. The consequence of inconsistency is closely related to the so-called paradoxes o f material implication that if a statement is false then it materially implies any statement whatever. Similarl y any argument with inconsistent premisses is valid regardless of its conclusion. In other words, an y statement whatever can be validly inferred from an inconsistent set of propositions. This result helps exp lain why consistency is so highly prized.

F. Quantification Theory

1. Singular Propositions 2. Quantification 3. Traditional Subject-Predicate Propositions 4. Proving Validity 5. Proving Invalidity 6. Asyllogistic Inference

Introduction: What is Quantification Theory? It is a method with which non-compound statements can be described and symbolize d in such a way that their inner logical structure will be revealed. This method uses additional symbolization to apply the rules of inference more w idely, in what is called the Logic of Predicates.

What is Predicate Logic? Predicate logic studies the relations between names, predicates, and quantifiers . Names: Refer to individual people, places and things. Ex. Khalifa, Middlebury, computer Predicates: Refer to properties of and relations between people, places, and thi ngs. Ex. is a professor. teaches at , etc. Quantifiers: Logical operators that reflect relations between subjects and predi cates. Ex. All and Some This

Example: Khalifa is a professor. Khalifa works at Middlebury. Everybody who is a professor and works at Middlebury teaches at Middlebury. So Khalifa teaches at Middlebury.

Singular Propositions: The most basic proposition in predicate logic is one in which something is predi cated of an individual name(s). Ex. Socrates is a man. Khalifa is a professor. Khalifa teaches at Middlebury. Names are represented as lowercase letters a through w. (a-w) s = Socrates; k = Khalifa; m = Middlebury Predicates are represented with capital letters. Some predicates are one-place predicates: M = is a man; P = is a professor. Others are n-place predicates: T = teaches at.

2. Quantification

Individuals, Predicates, and Quantifiers Quantification theory employs special symbols of four special sorts:

1. Individual constants ( a, b, c, etc. ) represent particular individual things A llison, Bill, or this car, for example.

2. Predicate constants ( F, G, H, etc. ) represent specific predicates free, greed y, or heavy, for example. So we use capital letters to symbolize attributes of the individual con stants Note: Always begin with the attribute symbol, then followed by the individual s ymbol. Writing an individual symbol directly after a predicate symbol signifies that th is thing has that property. Thus, using the above examples, Fa, Gb, and Hc would signify "Allison is free," "Bill is greedy," and "This car is heavy" respectively.

Individual variables ( x, y, z ) are employed to refer to any predicate generall y. They are used as place holders for individual constants.

3. Quantifiers are logical operators that signify the range of individuals to wh ich individual variables apply, either universal all - (x) or existential some - (.x).

Proper notation for singular propositions: PREDICATE, then NAME(S) = PROPOSITION. Socrates is a man: Ms Khalifa is a professor: Pk Khalifa teaches at Middlebury: Tkm With n-place predicates, order of names matters. Tmk means Middlebury teaches at Khalifa.

Ungrammatical to write a name followed by a predicate.

Propsitional Function: It is an expression that: 1. contains an individual variable and 2. becomes a statement when an individual constant is substituted for the indivi dual variable. When we have a predicate, then letters x through z, we have a propositional func tion. Example: Mx, Px, Txy A propositional function is not itself a proposition although it can become one by substitution. Without quantifiers, these are not grammatical. 1. Thing is mortal, Thing is professor, Thing teaches at other thing.

Quantifiers: Two kinds: Universal: represented either as (x): For all x Existential: .x: There is at least one x such that These are also not propositions by themselves. However, quantifiers plus propositional formulas/ functions are propositions. (x)Bx= Everything is beautiful. (.x)Px = Someone is a professor. (.x)Txm = Somebody teaches at Middlebury.

1. Universal Quantification General Proposition: All things are mortal. (Mx) Given any individual thing whatever, it is mortal. *Let s use x as our individual variable and use it to substitute both the pronoun -thing and its antecedent -it.] Thus: Given any x, x is mortal. Then using the notation for predicates, we substitute the attribute mortal with M . Thus: Given any x, Mx. The phrase Given any x is called the universal quantifier, which is customarily sy mbolized as (x)

We may now completely symbolize our general proposition as: (x) Mx

2. Existential Quantification: General Proposition: Something is beautiful. (Bx) There is at least one thing that is beautiful. *The word that is an antecedent referring back to the pronoun the individual variable x to both.+ thing. Let s substitute

Thus: There is at least one x such that x is beautiful. Then using the notation for predicates, we substitute the attribute beautiful wi th B. Thus: There is at least one x such that Bx. The phrase There is at least one x such that is called the existential quantifier, which is customarily symbolized as (.x) Thus, the complete symbolization is: (.x) Bx

Equivalences: ~(x)Fx . (.x)~Fx No one is friendly . Someone is not friendly. ~(.x)Fx . (x)~Fx There is not one person who is friendly . Everybody is not friendly. ~(x)~Fx . (.x)Fx

Not everybody is not friendly . Somebody is friendly. ~(.x)~Fx . (x)Fx There is not one person who is not friendly . Everybody is friendly.

Quantification of the A proposition The A proposition All things are mortal, asserts that if anything is a human then it is mortal. In other words, for any given thing x, if x is a human then x is mortal. Substituting the horsehoe symbol for if-then we get:

Given any x, x is human . x is mortal. (x) (Hx . Mx)

Quantification of the E proposition The E proposition, No humans are mortals asserts that if anything is human, then i t is not mortal. In other words, for any given thing x, if x is a human then x is not mortal. Thus,

Given any x, x is a human . x is not mortal. (x) (Hx . ~Mx)

Quantification of the I proposition The I proposition, Some humans are mortals asserts that there is at least one thin g that is a human and is a mortal. In other words, there is at least one x such that x is a human and x i s a mortal. Thus,

There is at least one x, such that x is a human

x is mortal. (.x) (Hx

Mx)

Quantification of the O proposition The O proposition, Some humans are not mortals asserts that there is at least one thing that is a human and is not a mortal. In other words, there is at least one x such that x i s a human and x is not a mortal. Thus,

There is at least one x, such that x is a human

x is not mortal. (.x) (Hx

~Mx)

Put together, these symbols can be used to represent each of the four major form s of Categorical Proposition: 1. 2. 3. 4. (x)(Hx . (x)(Hx . (.x)(Hx (.x)(Hx Mx) corresponds to the A proposition, "All S are P." ~Mx) corresponds to the E proposition, "No S are P." Mx) corresponds to the I proposition, "Some S are P." ~Mx) corresponds to the O proposition, "Some S are not P."

Examples: Sparrows are not mammals (x)(Sx . ~Mx) Nurses are always considerate (x)(Nx . Cx) No boy scout ever cheats. (x)(Bx . ~Cx)

4. Proving Validity

1. Universal Instantiation a rule of inference that permits the valid inference of any substitution instance of a propositional function from its universal quantification. UI: (x) (.x) .: .v where v is any individual symbol

All humans are mortal. Socrates is human. Therefore, Socrates is mortal.

1. (x) (Hx . Mx) 2. Hs .: Ms 3. Hs . Ms 1, UI 4. Ms 3, 2, M.P. Modus Ponens - p . q, p, .: q

2. Universal Generalization a rule of inference that permits the valid inference of a universally quantified expression from an expression that is given as true of any arbitraril y selected individual. UG: .y .: (x) (.x) where y All humans are mortal. All Greeks are human. Therefore, all Greeks are mortal. 1. (x) (Hx . Mx) 2. (x) (Gx . Hx) .: (x) (Gx . Mx) 3. Hy . My 1, UI 4. Gy . Hy 2, UI 5. Gy . My 4,3, H.S. 6. (x) (Gx . Mx) 5, UG Hypothetical Syllogism p . q, q . r, .: p . r is any arbitrarily selected individual

3. Existential Instantiation a rule of inference that permits (with restrictions ) the valid inference of the truth of a substitution instance from the existential quantification of a propos itional function. EI: (.x) (.x) .: .v where v 1. (x) (Cx . Vx) is any individual constant

2. (.x) (Hx .: (.x) (Hx 3. Ha

Cx) Vx)

Ca 2, EI

4. Ca . Va 1, UI 5. Ca Ha 3, Com.

6. Ca 5, Simp. 7. Va 4,6, M.P. 8. Ha 3, Simp. 9. Ha Va 8,7 Conj.

10. use of existential generalization

Hypothetical Syllogism (H.S.) Absorption (Abs.) p . q p . q q . r .: p . (p .: p . r Disjunctive Syllogism (D.S.) Addition (Add.) p v q p ~p .: p v q .: q Constructive Dilemma (C.D.) (p . q) (r. s) q)

p v r .: q v s

6. Asyllogistic Inference

- arguments containing one or more propositions more logically complicated than the standard A, E, I, O propositions. - they cannot be reduced to a standard form categorical syllogism and evaluating them requires more logic than was traditionally used in testing categorical syllogisms.

Examples: Apples and oranges are delicious and nutritious. (x) [ (Ax V Ox) . (Dx Nx)+

A gladiator wins if and only if he is lucky. (x) Gx . (Wx = Lx)

To achieve success, one must work hard if one goes into business, or study cont inuously if one enters a profession. (x) {Ax . [ (Bx . Wx) (Px . Sx)] }

V. INDUCTIVE REASONING

A. Inductive Generalizations (Induction by Simple Enumeration)

B. Analogy and Probable Inference

1. Analogy 2. Appraising Analogical Arguments 3. Refutation by Logical Analogy

C. Causality D. Probability

V. INDUCTIVE REASONING A. Inductive Generalizations (Induction by Simple Enumeration)

INDUCTIVE REASONING: - reasoning from detailed facts to general principles

- An inductive generalization takes a sample of a population and makes a conclu sion regarding the entire population. Inductive Reasoning is reasoning from experience, sense perceptions, and observa tions to form a conclusion. This is the most common form of reasoning that humans partake in. Th e most common type of inductive reasoning is in forming an expectation of what will happen in the n ear future, based upon past observations. It draws inferences from observations in order to make generalizations. FOUR STAGES OF INFERENCE: 1. 2. 3. en 4. Observation: collect facts, without bias. Analysis: classify the facts, identifying patterns o of regularity. Inference: From the patterns, infer generalizations about the relations betwe the facts. Confirmation: Testing the inference through further observation.

THREE FORMS OF INDUCTIVE ARGUMENT: 1. Part-to-whole: where the whole is assumed to be like individual parts (only b igger). 2. Extrapolations: where areas beyond the area of study are assumed to be like t he studied area. 3. Predictions: where the future is assumed to be like the past. INDUCTIVE GENERALIZATION: A generalization (more accurately, an inductive generalization) proceeds from a premise about a sample to a conclusion about the population. The proportion Q of the sample has attribute A. Therefore: The proportion Q of the population has attribute A. Example: There are 20 balls either black or white in an urn. To estimate their respective num bers, you draw a sample of four balls and find that three are black and one is white. A go od inductive generalization would be that there are 15 black, and five white, balls in the ur n. How much the premises support the conclusion depends upon (a) the number in the sample group, (b) the number in the population, and (c) the degree to which the sample represents the population (which may be achieved by taking a random sample). The hasty generali

zation and the biased sample are generalization fallacies.

INDUCTION BY SIMPLE ENUMERATION: - Induction by enumeration, also called enumerative induction, is thought to be the fundamental form of inductive reasoning. It is non-demonstrative, and the truth of the concl usion is not guaranteed.

In logic, an enumerative induction is an argument of the type simple inductive s chema. Induction by simple enumeration is no proof of causality. It only suggests a pot ential causal relation between two items. Francis Bacon wrote in The Advancement of Learning "The induction that proceeds by simple enumeration is childish." Example: When we observe that A 1 , A 2 , A 3 , A n , all have a property B, and that no As have been found not to have B, we infer that all As have the property of B.

B. Analogy and Probable Inference Analogical Reasoning The simplest variety of inductive reasoning is argument by analogy, which takes note of the fact that two or more things are similar in some respects and concludes that they are prob ably also similar in some further respect. Not every analogy is an argument; we frequently use such compar isons simply to explain or illustrate what we mean. But arguments by analogy are common, too. Suppose, for example, that I am thinking about buying a new car. I'm very likely to speak with other people who have recently bought new cars, noting their experiences with various makes, models, and dealers. If I discover that three of my friends have recently bought Geo Prizms from Burg and that all three have been delighted with their purchases, then I will conclude by analogy that i f I buy a Geo Prizm from Burg, I will be delighted, too.

Evaluating Analogies Of course, this argument is not deductively valid; it is always possible that my new car may turn out to be an exception. But there are several considerations that clearly matter in det ermining the relative strength or weakness of my inductive inference: 1. Number of instances. If five friends instead of three report their satisfacti on with the model I intend to buy, that tends to make it even more likely that I will be satisfied, too. In general, more instances strengthen an analogy; fewer weaken it. 2. Instance variety. If my three friends bought their Prizms from three differen t dealers but were all delighted, then my conclusion is somewhat more likely to be true, no matter wher e I decide to buy mine. In general, the more variety there is among the instances, the stronger th e analogical argument becomes. 3. Number of similarities. If my new purchase is not only the same make and mode l from the same dealer but also has the same engine, then my conclusion is more likely to be tru e. In general, the more similarities there are between the instances and my conclusion, the better for the analogical argument. 4. Relevance. Of course, the criteria we're considering apply only if the matter s with which they are concerned are relevant to the argument. Ordinarily, for example, we would assume that the day of the week on which a car was purchased is irrelevant to a buyer's satisfaction wi th it. But relevance is not something about which we can be terribly precise; it is always possible in p rinciple to tell a story in the context of which anything may turn out to be relevant. So we just have to use our best judgment in deciding whether or not some respect deserves to be considered. 5. Number of dissimilarities. If my friends all bought Geos with automatic trans missions and I plan to buy a Geo with a standard transmission, then the conclusion that I will be delig hted with my purchase is a little less likely to be true. In general, the fewer dissimilariti es between instances and conclusion, the better an analogical argument is. 6. Modesty of conclusion. If all three of my friends were delighted with their a uto purchases but I conclude only that I will be satisfied with mine, then this relatively modest co nclusion is more likely to be true. In general, arguments by analogy are improved when their conclusions are modest with respect to their premises.

2. Appraising Analogical Arguments 3. Refutation by Logical Analogy

APPRAISING ANALOGICAL ARGUMENTS: 1. Number of entities a. Larger the number of entities the stronger the argument i. No ration between number and probability

2. Variety of the instances in the premises a. More dissimilar instances mentioned, the stronger the argument

3. Number of similar respects a. Greater of number of respects of similarity of entity in conclusion to entiti es in premises, stronger the argument

4. 5. 6. 7.

Relevance Dis-analogies Claim that the conclusion makes More modest the claim, stronger the argument

REFUTATION BY LOGICAL ANALOGY: A deductive argumentcan be refuted (i.e., shown to be invalid, hence unsound by: stating a second argument that has all three of the following features: (1) the same form as the first argument; (2) true premises; and (3) a false conclusion. This is called refutation by logical analogy (similar) forms. because the arguments have analogous

What is the rationale for this procedure? The first thing to understand is that validity is a formal or structural property of arguments. To say that an argument is valid is to say nothing about whether its constituent propositions are true. What it says is that if the premises are true, then the c onclusion is true. A valid argument is such that it is logically impossible for its premise(s) to be true a nd its conclusion false. Validity is a relation between premise(s) and conclusion, not a property of premise(s) or co nclusion. A valid argument is one that preserves truth. We value validity because and only because we value tru th. It is a means to our end. Since validity concerns only the form of an argument, if two arguments have the same form and one of them is invalid, the other is invalid. (In other words, two arguments with the same f orm are either both valid or both invalid.) Suppose we are wondering whether a particular argument, X, is val id. One way to find out (and here I repeat what I said earlier) is to try to construct another argument of the same form as X that has true premises and a false conclusion. Suppose we can do this. Then, given the de finition of valid argument, the second (constructed) argument is invalid, for (by definition) no va lid argument has true premises and a false conclusion. But if the second argument is invalid and has t he same form as X, then X is invalid. Example: Suppose I wish to refute the argument that, since no conservatives are liberals and all religious people are conservatives, all religious people are liberals. This argument call it A1 has the following form:

1. No C is L. 2. All R is C. Therefore, 3. All R is L.

Let me substitute terms for the letters rue and 3 false. Here is A2: 1a. No triangles are squares.

C,

L,

and R

in such a way as to make 1 and 2 t

2a. All three-sided figures are triangles. Therefore, 3a. All three-sided figures are squares Premises 1a and 2a are true (in fact, necessarily so), but 3a is false (again, n ecessarily so). What this shows is that A2 is invalid, for, by definition, no valid argument has true premises and a false conclusion. But if A2 is invalid and A2 has the same form as A1, then A1 is invalid. The refutation is co mplete.

VI. FALLACIES A. What is a Fallacy? The term fallacy is derived from the Latin word fallo . It means, etymologically eive. A fallacy is a deceptive argument. It is an argument that seems to be conclusive but it is actually not conclusive. Its sequence seems to be valid but it is actually invalid. Also, its premises may seem to be true but are actually false. Advantages of knowing Fallacies: 1. The unique ability to call a fallacy by name will give a person a greater adv antage over an opponent in debate and argumentation. 2. The valid forms of inference can best be shown and explained by contrasting t hem with invalid ones. 3. Mastery in the recognition of fallacies will enable one to apply the principl es of logic to everything one reads or hears. 4. the study of fallacies will provide the occasion for a review of much of the lessons concerning rudiments of logic. 5. Fallacies serve as danger signals to warn us of injudicious reasoning. Famili arity with these errors in logic can assist us from being deceived by them. B. Formal Fallacies I dec

1. Fallacies in Categorical Syllogisms

a. Four Terms b. Undistributed Middle Term c. Illicit Major Term d. Illicit Minor Term e. Negative Premises f. Particular Premises

Logical fallacies are errors in inferring. They are not merely errors of interpr etation. Formal fallacies - those that arise from the violations of the rules of syllogis m. Any violation of these rules gives rise to a mistake in inference. A. Fallacy of 4 terms: Examples: Filipinos are oriental; Frenchmen are Europeans; Filipinos are Europeans.

Dog is a noun; A dog eats meat; A noun eats meat.

B. Fallacy of undistributed middle term- the rule of syllogism requires that the middle term must be distributed at least once. A term is distributed when it is used universally. Example: Filipinos are orientals; Japanese are orientals; Filipinos are Japanese.

In the example, the middle term orientals is not distributed either in the major premise or in the minor premise. Hence, the fallacy.

C. The Fallacy of Illicit Major Term arises when the major term is distributed i n the conclusion but it was not distributed in the major premise. Example: Filipinos are heroic people. Americans are not Filipinos; Americans are not heroic people.

In the example, the major term heroic people is not distributed in the major premi se. The major premise is distributed in the conclusion because it used negatively. Hence, the fallacy.

d. The Fallacy of Illicit Minor Term- a violation of the rule that the minor ter m should not be distributed in the conclusion when it was not distributed in the minor premise.

Example: All representatives are at least 25 years of age; All representatives are voters; All voters are at least 25 years of age.

The minor term voters is distributed in the conclusion when it was not distribut ed in the minor premise.

e. The Fallacy of Negative Premises occurs when the premises are both negative. Example: Cows are not dogs. Horses are not dogs. (no conclusion).

2. Fallacies in Hypothetical Syllogisms (Modus Ponens and Modus Tollens) a. Denial of Antecedent b. Affirmation of Consequent

Formal Fallacies in hypothetical syllogisms: Hypothetical syllogisms are different from standard syllogisms and thus have th eir own rules. In a hypothetical syllogism the first premise (or major proposition) presents an unce rtain condition ("if A, then B") or a problem ("either A or B"; "S and T cannot both be true") which must then be properly resolved by the second premise so that a valid conclusion can follow. The resolution of the prob lem is always in the form of affirmation or denial. 1. Modus Ponens: In classical logic, modus ponendo ponens (Latin for the way that affirms by affi rming; often abbreviated to MP or modus ponens) If P implies Q and P is true, then Q is true.

Example: If it's raining, I'll meet you at the movie theater. It's raining. Therefore, I'll meet you at the movie theater.

2. Modus Tollens: In classical logic, modus tollens (or modus tollendo tollens) (Latin for "the wa y that denies by denying") Modus tollendo tollens, usually simply called modus tollens or MT is a valid arg ument form in logic. It is also known as "denying the consequent". The form of modus tollens is: "If P, then Q. Not Q. Therefore, not P." It may al so be written as: P . Q, Q P Example: If Sam was born in Canada, then he is Canadian. Sam is not Canadian. Therefore, Sam was not born in Canada.

A. Fallacy of denying the antecedent- committed when the minor premise denies th e antecedent of the major premise. (Antecedent is one which states the condition or limitation.) Denying the antecedent, sometimes also called inverse error, is a formal fallacy , committed by reasoning in the Form: If p then q. Not p. Therefore, not q.

Example: If Pedro marries Juana, she will be happy; Pedro will not marry Juana; Juana will not be happy.

When the antecedent is denied in Pedro will not marry Juana, it does not necessari ly follow that Juana will not be happy. On the other hand, if the antecedent is affirmed, the conclusi on, Juana will be happy, follows logically. B. Fallacy Affirming the Consequent arises when the minor premise affirms the co nsequent. Affirming the consequent, sometimes called converse error, is a formal fallacy, committed by reasoning in the form: If P, then Q. Q. Therefore, P.

Example: If David graduated with honors, he will be appointed to the faculty; David has been appointed to the faculty; David graduated with honors.

When the consequent is affirmed, David has been appointed to the faculty, it does not necessarily follow that David graduated with honors. The major premise does not say that David would not be appointed unless he graduated with honors. 3. Fallacies in Disjunctive Syllogisms a. Missing Disjuncts b. Nonexclusivity

Conditional Proposition

if-then

statement.

Example: If a man dies, then he is a mortal being. Disjunctive proposition are either-or statement.

Example: Either a man is polite or impolite. Conjunctive Proposition consist of two statements conjoined by the word and .

Example: The tree cannot be mahogany and an ipil-ipil at the same time. Disjunctive Syllogisms: in classical logic, is historically known as modus tollendo ponens it is a type of hypothetical syllogism whose major premise is a disjunctive prop osition while its minor premise and conclusion are categorical propositions. Example: Major Premise: The man is either honest or dishonest. (Disjunctive prop osition) Minor Premise: He is honest. (Categorical proposition) Conclusion: Therefore, he is dishonest.(Categorical proposition)

Kinds of Disjunctive Syllogisms: Classified as: 1. Exclusive Disjunction - The component parts of a proper disjunctive proposition can neither be true n or false together (RULE: if one is true, the other must be false; if one must false, the other mu st be true) - Has two valid moods (ponendo tollens & tollendo ponens 2. Inclusive Disjunction - Has one valid mood (ponendo tollens) and one invalid mood (tollendo ponens) Deduce two valid moods 1. Ponendo Tollens (Accept-Reject) RULE: If one disjunct is accepted in the minor premise, the other disjunct must be rejected in the conclusion. . Example: (Accept-Reject) The man is either honest or dishonest. He is honest. Therefore, he is not dishonest. The man is either honest or dishonest. He is dishonest. Therefore, he is not honest. Example: (Reject-Accept) The man is either honest or dishonest. He is NOT honest. Therefore, he is dishonest. The man is either honest or dishonest. He is NOT dishonest.

Therefore, he is honest.

2. Tollendo Ponens (Reject-Accept) RULE: if one disjunct is rejected in the minor premise, the other disjunct must be accepted in the conclusion. Four possible forms of two valid moods: 1. Both parts may be affirmative. 2. Both parts may be negative. 3. The first part may be affirmative, the second may be negative. 4. The first part may be negative, the second may be affirmative. Form 1. Both parts may be affirmative. Tollendo Ponens It is either A or B. It is not A. Therefore, it is B.

Example: A person is either educated or uneducated. He is not uneducated. Therefore, he is educated.

Ponendo Tollens It is either A or B. It is A. Therefore, it is not B.

Example: A person is either educated or uneducated.

He is educated. Therefore, he is not uneducated.

Form 2. Both parts may be negative.

Tollendo Ponens It is either not A or not B. It is A. (it is not not A) Therefore, it is not B.

Example: A being is either not immortal or not mortal. It is immortal. Therefore, it is not mortal.

Ponendo Tollens It is either not A or not B. It is not A. Therefore, it is B. (it is not not B)

Example: A being is either not immortal or not mortal. It is not immortal.

Therefore, it is mortal.

Form 3. The first part may be affirmative second may be negative.

Tollendo Ponens It is either A or not B. It is not A. (it is not not A) Therefore, it is not B.

Example: Either he is unmarried or not single. He is not unmarried. Therefore, he is not single.

Ponendo Tollens It is either A or not B. It is A. Therefore, it is B. (it is not not B)

Example: Either he is unmarried or not single. He is unmarried. Therefore, he is single.

Forms 4. The first part may be negative, the second may be affirmative.

Вам также может понравиться